AAMC QPack Bio2 [Ext]

First thing we want to do is look at Figure 1 from the passage and compare the rates of actin subunit addition of both ends. We want to find the concentration of free actin where the plus end of the microfilament grows faster than the minus end. Concentration of free actin is shown on the X-axis. One thing I want to note here. At very low concentrations, meaning 1 micromolar or below, we have a loss of actin subunits on both the plus and minus end. Only after crossing the 1 micromolar threshold do we start getting addition of actin subunits on the plus end. The solid line represents the plus end, and it has a much larger slope. That’s because the plus end of the microfilament grows faster. Dotted line represents the minus end and has the smaller slope, meaning it grows more slowly. That means at every concentration above 1 micromolar we have the plus end of the microfilament growing, and growing faster than the minus end.
This answer choice corresponds to a net change of zero to the plus end. The rate of loss is equal to the rate of addition, so there’s no net change. We’re not seeing the plus end growing.
This answer choice is partially true. We do have the plus end of the microfilament growing faster than the minus end between these concentrations. But this answer choice says “Only.” We’re not limited to just these concentrations. Answer choice B is an incomplete answer, but it’s better than option A.
This answer choice is consistent with our breakdown of the question. We said that at every concentration above 1 micromolar we have the plus end of the microfilament growing, and growing faster than the minus end. This is superior to answer choices A and B.
This is not true because at a concentration under 1 micromolar, there is a loss of actin subunits on both the plus and minus ends. This contradicts our breakdown and Figure 1 from the passage. We’re left with our correct answer: answer choice C.
I have to make sure to be careful with the wording. I want to explain why microfilament lengths do NOT change when the sarcomere shortens in a muscle contraction. We have our image of the muscle sarcomere in the question stem, with labels. Myosin is also known as thick filament. Thin filament is actin, and also troponin and tropomyosin. The A band spans the length of the thick filament and also contains thin filaments.
We have the Z-line labeled which defines the boundaries of each sarcomere. I-band is the region with only thin filaments, and H-zone contains only thick filaments. When the sarcomere shortens, the H-zone, I-band, the distance between Z-lines all become smaller. The minus sign of the microfilament is anchored in that Z-line. Even though we have different parts of the sarcomere shortening, the microfilament is anchored in the Z-line, and only the distance between microfilaments will decrease.
First part of this answer choice is consistent with our breakdown. Microfilaments are capped by Z-lines on the minus end. But the microfilament length isn’t actually varying on the plus side either. The distance between microfilaments varies we said, not the length of the microfilaments. Both sides are capped to prevent addition or subtraction of actin subunits. An actin subunit concentration above 1 micromolar would normally mean addition to the plus end, but we mentioned there are no actin subunits being added to that end. Second part of the answer choice isn’t a great explanation for why length doesn’t change.
This answer choice matches our breakdown, and what we just said when we went through answer choice A. We have both ends of the microfilaments capped, which prevents microfilament length from changing. Answer choice B is superior to answer choice A.
This answer choice is out of scope. It’s not explaining why microfilament lengths don’t change. At a concentration above 4 micromolar, we would expect both the plus and minus end of the microfilament to add actin subunits. Not what’s happening here.
This is the opposite of our breakdown. We expect both ends to be capped, and no growth. The only way answer choice D would work is if neither side was capped, and we had treadmilling. Not the case here because the minus end is anchored in the z-line. We’re left with our correct answer, answer choice B.
We can reference the passage to find where the author mentions force generation specifically. The passage says “Researchers suspect that microfilaments can generate force, even in the absence of myosin, by elongating and pushing against a structure such as the plasma membrane.”
Normally microfilaments help generate forces used in cellular contraction and basic cell movements. This is done in association with myosin. In this case, the author mentions force can be generated even in the absence of myosin. That force is generated by elongating and pushing against a structure. Elongation happens with the addition of subunits to the ends of the microfilaments.
This answer choice can explain the theory of force generation. First thing we’ll note is cytochalasins bind to the positive end of a microfilament and prevents the addition of actin subunits to that end. So, this poison stops amoeboid movement, and stops elongation. This is a good option for now.
This answer choice insinuates that mitosis leads to cell movement. There’s no mention of force generation and mitosis together in the passage. Answer choice A directly answered the question being asked, and answer choice B isn’t related to the passage.
This answer choice is similar to answer choice B. There’s no mention of troponin in the passage. We did just mention in our previous question that thin filament is made of actin, and also troponin and tropomyosin. That’s the only mention we made, and there’s no way to conclude that additional troponin supports the theory.
This is a third straight answer choice that is not discussed in the passage. We didn’t discuss the medium in which microfilaments are found, or discuss viscosity influencing subunit addition rate. Best option here was the first one: answer choice A.
We’ll use Figure 1. We want to know about treadmilling, or when the rate of subunit addition at the plus end equals the rate of subunit loss at the minus end. The key to this question is looking at both ends at the same time. If we look at a concentration of 1.5 micromolar, we see rate of subunit addition at the plus end is 2 units greater than zero and we see the rate of subunit loss at the minus end is 2 units less than zero. That means there is a net change of zero, or treadmilling. Let’s look for a concentration of 1.5 micromolar in the answer choices.
This is a numerical answer with no rounding or estimating. We solved for a value of 1.5 micromolar. We can compare all of our answer choices at once. Our correct answer is going to be answer choice C.
One more time we’ll rely on Figure 1. We want to know the concentration of free actin at which there is exclusively net loss of actin subunits. That means we have a value less than zero on the Y-axis for both the plus and minus ends. We see that’s true for the minus end at any concentration below 4 micromolar; for the plus end, that’s true at any concentration below 1 micromolar. The key here is looking at both rates and not just one. That means at concentrations below 1 micromolar, both the plus and minus ends of the microfilament experience a net loss of subunits.
This was another numerical answer with no rounding or estimating. We solved for an exact answer, which was any concentration below 1 micromolar. We can compare all of our answer choices at once. Answer choice A is the only answer that matches that value exactly.

I am going back to the passage to pinpoint the aspect of experiment 1 that shows cell-to-cell communication determines cell fate. There are a lot of smaller words and numbers we’ll be referencing, so it can be beneficial to have the passage in front of you as you go through the solutions.

We can focus on the 2nd paragraph here. It says “The cultured AB cells produced neurons and skin, but no muscle, whereas the cultured P1 cells gave rise to all of the tissues produced by P1 cells of an intact embryo.”
What’s the takeaway here? Cultured AB cells produced only neurons and skin, but no muscle. The cultured P1 cells gave rise to all of the tissues normally produced by P1 cells of an intact embryo. What did this tell us? The P1 cells are acting the same, regardless of the presence of AB. But AB produces a different result when isolated from P1. That likely means AB is typically dependent on communication with P1 to produce normally. P1 is the same, regardless of the presence of AB.

This answer choice is consistent with my breakdown of the question. I mentioned AB cells normally produce neurons, skin, and muscle. But the isolated AB cell only produced neurons and skin. I said this is because AB is typically dependent on communication with P1. This is a good answer choice to start.
This answer choice is technically a correct statement, but does it answer the question being asked? Because we can’t distinguish between the isolated P1 cell and the P1 cell in the intact embryo, we can’t support the hypothesis that cell-to-cell communication is involved in the determination of cell fate. This answer choice is out of scope and doesn’t answer the question being asked.
This answer choice is inconsistent with the breakdown of the question and what we were told in the passage. Isolating blastomeres at the two-cell stage led us to find that AB cells are dependent on communication with P1 to produce normally. We can stick with our superior answer here, answer choice A.
This answer choice is similar to answer choice B. The statement is technically correct, but it doesn’t actually answer the specific question being asked. There are multiple blastomeres that can produce neurons and muscle tissue, but that doesn’t support the hypothesis that cell-to-cell communication is involved in the determination of cell fate. Once we get rid of option D, we’re left with our correct answer, answer choice A: The fate of an isolated AB cell differs from that of an AB cell in an intact embryo
Let’s go back to Experiment 1-this is going to be similar to our previous question. We’re going to focus on what we focused on for Question 6. Cultured AB cells produced only neurons and skin, but no muscle. The cultured P1 cells gave rise to all of the tissues normally produced by P1 cells of an intact embryo.
That means the P1 cells are acting the same, regardless of the presence of AB. But AB produces a different result when isolated from P1. That means AB is dependent on communication with P1 to produce normally. We expect the direction of signaling of a two-cell embryo is P1 -> AB
This is the opposite of what I said during the breakdown of the question. We expect AB is dependent on communication with P1, not the other way around.
This answer choice matches what I said during the initial breakdown, and also what we covered in Question 6. AB is dependent on communication with P1 to produce normally. We expect the direction of signaling of a two-cell embryo is P1  AB. This answer choice is superior to answer choice A.
This answer choice is out of scope. We don’t cover P2 in Experiment 1. We already have an answer choice that matches the passage and predicted answer exactly, so we’re sticking with answer choice B for now.
Answer choice D is similar to answer choice C. There is no mention of the one-celled zygote in Experiment 1. Instead we focused on separating the cells of a two-cell embryo and focusing on AB cells and P1 cells. We’re left with our correct answer, answer choice B.
Previously we used Experiment 1, now we’re using Experiment 2. From the passage, we know we normally have AB cells produce neurons, skin, and muscle. When these cells were treated with cycloheximide, which is an inhibitor of translation, the cells only produced neurons and skin; no muscle was produced.
When the cells were treated with actinomycin D, which is an inhibitor of transcription, the cells produced neurons, skin, and muscles, which is what’s normally produced by AB cells. Transcription produces mRNA, and translation produces proteins. Our correct answer is going to focus on translation and proteins. Inhibiting protein synthesis affected the normal production of the two-cell embryo.
Neither of our inhibitors directly inhibit the production of DNA. That means if DNA were involved, we’d expect the same results in both sets of AB cells. That’s not the case here. The cells that were incubated in the presence of cycloheximide produced only neurons and skin, and no muscle.
This would be the correct answer if the results of Experiment 2 were switched. The inhibitor that affected translation was the one that caused no muscle to be produced. It was not the inhibitor that affected transcription. This is a better answer than answer choice A, because if messenger DNA were the correct answer, we’d at least be able to figure it out through experimentation. We can eliminate answer choice A.
This is similar to answer choice A. rRNA acts as the building blocks of ribosomes, but isn’t the end product of transcription or translation. There’s no indication in the experiment that rRNA is involved in signaling interaction at the two-cell stage. Answer choice B is still a superior answer choice.
This answer choice matches our breakdown of the question. When cells are treated with cycloheximide, cells only produced neurons and skin, and no muscle. Cycloheximide inhibits translation, which ultimately produces proteins. Answer choice D, protein, is going to be our best and correct answer.

We’ll reference Experiment 3. We’ll look for a pair of cells where we have gut differentiation and specification. Also, note how passage and visual-heavy these questions are. This is 4 straight questions where we’re getting details from the passage. Make sure to understand the passage well in your initial read through.

We have Experiment 3 and Figure 2 shown here. Shaded circles say gut differentiation, and the unshaded circles say no gut differentiation. We see the 3rd set down, the EMS and P2 cell combination. That’s where gut differentiation results at all times. That’s the indicator that communication between EMS and P2 results in gut differentiation.

This answer matches our breakdown of the question exactly. We have gut differentiation at every time on the timeline when we have EMS and P2 together. We can keep this answer choice for now, but we’re going to remain unbiased and quickly go through our other answers. There is a good chance this is going to be our correct answer because matches my breakdown exactly.
This answer choice contradicts what we saw in Figure 2, and our breakdown. If we look at the timeline, we don’t have the same differentiation we see with P2 and EMS. We can eliminate answer choice B.
This answer choice contradicts what we saw in Figure 2, and our breakdown. Similar to answer choice B, we don’t have gut differentiation the same way we do for EMS and P2. We can eliminate answer choice C.
This answer choice is similar to answer choices B and C. It contradicts what we saw in Figure 2 and the breakdown. We can stick to our correct answer, answer choice A: P2 and EMS.
We’re tracking the sequence by which gut differentiation happens. From zygote to P1, and ultimately to gut.
Looking at the Figure, the predicted sequence is Zygote > P1 > EMS > E
This doesn’t match what I said in breaking down the question.
This matches our predicted sequence exactly.
This doesn’t match what I said in breaking down the question. It wrongly includes an additional P2.
This doesn’t match what I said in breaking down the question.

Essentially, this boils to knowing why the two can have different genes. This answer is going to come from the passage, and specifically the first part of the passage where the author talks about the building of a cytoplasmic bridge and the movement of plasmid DNA through the pilus.

Let’s look at the bottom paragraph here that says, “Conjugation occurs when the plasmid genes of one bacterium direct the building of a cytoplasmic bridge, or sexpilus, between that organism and a bacterium lacking such genes.”
First thing we want to note, is plasmid genes of bacterium direct the building of sexpilus. These are the genes we’re focused on in the question stem. What do we know about plasmids from earlier in the passage? “Because plasmids are not attached to the cell membrane, they may not be equally distributed among daughter cells.” This is as simple as combining what we learned from these two sentences to answer our question. Why might a daughter cell not have the same genes for sex pilus construction? Because the daughter cells did not get the plasmid for sex pilus construction because of how the genes were distributed.

This answer choice is unlikely. We don’t expect chromosomes to not be replicated completely. That’s just not a common thing. Even given that small chance, this answer could theoretically be correct if the genes that direct the building of the cytoplasmic bridge were chromosomal genes. We know sex pilus is directed by plasmid genes instead. This answer choice is unlikely.
This is another unlikely situation, but even if this situation happened, we said the sex pilus is directed by plasmid genes, not chromosomal genes. This is similar to answer choice A where neither is likely or factually correct.
This answer choice matches what I said in my breakdown of the question, and what the author mentions in the passage. Plasmids are not attached to the cell membrane, and may not be equally distributed to new daughter cells. Answer choice C is the best option so far.
This answer choice is implying bacteria have lysosomes, but that is not true. Lysosomes are membrane-bound organelles, and we typically only see them in animal cells. We can eliminate answer choice D for being unreasonable because we don’t have any bacterial lysosomes. We can stick with the correct answer, answer choice C: copies of the plasmid containing the genes were not equally distributed to the new daughter cells
For this question, we’ll have to explain how resistance to antibiotics in bacteria works. Quick overview of resistance: how are bacteria able to acquire antibiotic resistance? There’re a few reasons. The short generation time, random mutations, and genetic recombination allow bacteria to have a higher likelihood of acquiring antibiotic resistance. Note: the patient has only a few E. coli cells that remain in the colon. Let’s use this information and see if any of the answer choices are consistent with this thinking.
This answer choice is possible, but it’s unlikely. We want to know the most likely reason why these few E. coli cells are present. Antibiotics are not a cause of mutations. If this answer choice instead mentioned undergoing conjugation with antibiotic-resistant cells, this answer would be more likely to be correct. We’re not confident in the antibiotics causing mutations.
This answer choice is factually incorrect. Bacteria don’t work the same way as animals where they can develop an immune reaction to pathogens. This answer choice is also unlikely. We can still keep answer choice A as our best answer, because it’s more plausible, even if it’s not likely.
This answer choice is out of scope. We’re focused on the E. coli cells, not the patient’s colon cells in this question. Even if this were factually true, it doesn’t answer the specific question being asked. We can eliminate answer choice C.
This answer choice is possible, and it’s likely. Mutations can occur randomly. And when there are antibiotic-resistant bacteria, these bacteria tend to thrive. And the nonresistant bacteria are killed off. Answer choice D is a better answer than answer choice A
From reading the passage, recall that the bacteria are resistant to ampicillin and kanamycin. It’s likely the patient has a strain of the bacteria that is now resistant to the antibiotics. Plasmids can contain genes that provide resistance to specific antibiotics (like ampicillin and kanamycin). It’s likely that’s what happened here. That resistance is result of conjugation with resistant cells.
This answer choice is almost verbatim what we said during our readthrough and what we just mentioned as we went through our breakdown of the question. The bacteria that aren’t coexisting with the antibiotic resistant bacteria are clearly affected by antibiotics. That’s clear because the coworker was treated successfully, but the bacteria strain survived in the patient because it underwent conjugation with cells of resistant E. coli.
We actually touched on this in our previous question. Antibiotics themselves are not going to induce mutations in the bacteria. Instead, the bacteria underwent conjugation with the cells of resistant E. coli. We can eliminate this answer choice because the reasoning is factually incorrect.
The bacteria are not going to change the speed at which they reproduce. The strain survived because it adapted, not because it was reproducing more rapidly. This is another factually incorrect answer choice.
This is similar to answer choice C. The strain did not modify its metabolism to adapt to the new environment. The only difference we’re seeing is the strain underwent conjugation with resistant cells. That allows the strain to not be killed off when treated with ampicillin and kanamycin. The strain didn’t alter its metabolism in response to these bacteria. This answer choice is contradicting what I touched on in my breakdown of the question and it’s contradicting information from the passage. Answer choice A is our best option here.
We’re going to think through the effects of a ruptured appendix, and how that may cause infection. The test-maker gives us a clue in the question stem and tells us the patient had a bacterial infection. The issue with an appendix rupture is that bacteria the fine when it’s in the appendix or moving between the appendix and colon. In fact, we read in the passage: that same bacteria benefits humans by breaking down food and producing vitamins. Ultimately, the bacterial infection was problematic because of the bacteria infecting other organs. New research does suggest the appendix might be home to what we consider “good” bacteria, but we’re not focused on that for this question. We’re focused only on what’s in the passage, what’s asked in the question stem, and the topics listed on AAMC’s content outline.
This answer choice is unreasonable. The patient didn’t contract M. tuberculosis until after leaving the hospital from his initial stay. The ruptured appendix happened before tuberculosis, so initial reaction is we’re not liking this answer choice.
This answer choice is more plausible, and in fact likely happened. When the appendix ruptures, E. coli are going to enter other organs, like we said. The problem here is, E. coli inhabit the colon. The author mentions in the first paragraph that E. coli living in the colon actually have benefits, so this may not lead to bacterial infection. Let’s keep answer choice B for now, and eliminate answer choice A because it was unreasonable.
This answer choice sounds like it could be problematic. The author explicitly mentions E coli inhabit the colon. But there’s no mention of E. coli in the abdominal cavity being a good, or even a normal thing. Like we said in our breakdown, bacterial infection is problematic because bacteria infect organs where they’re not normally found. We can keep answer choice C for being a superior answer choice to answer choice B.
This is similar to answer choice B. We expect E. coli in the appendix and colon, and the bacteria can move between the two. We also expect if the appendix ruptures, E. coli would more likely move into the colon, rather than the other way around. The best answer here is still going to be answer choice C, because E. coli would enter the abdominal cavity, which is not normally supposed to happen.
We’re essentially looking at the fitness of the antibiotic-resistant E. coli cells against the other bacteria that are starting to colonize the patient’s colon. Hopefully the patient has better luck with their health going forward, but the reason E. coli cells would persist in the colon is similar to some aspects of natural selection. We expect the E. coli cells will stick around if they produce more offspring that can survive. We know the cells can survive in the colon, but it’s just a matter of reproducing.
This answer choice is straight to the point. In fact, I like the test-makers verbiage here better than my own breakdown of the question. The answer choice says viable descendants: meaning capable of living and reproducing. The fact that the antibiotic resistant E. coli cells produce more viable descendants than the other bacteria, the antibiotic-resistant bacteria will persist.
There’s no evidence to suggest producing more vitamin B12 would determine whether the antibiotic-resistant E. coli cells would persist. E. coli do benefit humans by producing vitamin B12, but there’s no reason to suggest that producing more vitamin B12 would allow one type of bacteria to survive over another. Answer choice A is still superior.
This is similar to answer choice B. Metabolizing glucose is something that happens with E. coli, but metabolizing glucose more quickly isn’t going to affect the persistence of the antibiotic-resistant bacteria. Ultimately the bacteria is going to have to be able to reproduce and produce more viable descendants. Answer A still remains the best option.
This sounds like a positive effect of the antibiotic-resistant E.coli, but doesn’t directly affect the survival of the bacteria. This is a battle between antibiotic-resistant E. coli cells and other bacteria. Ultimately, the antibiotic-resistant E. coli cells will only persist if they’re able to reproduce, and there are more viable descendants than the other bacteria. We’re left with our correct answer, answer choice A.
First thing we want to do here, as with any discrete question, is try and call to mind any relevant content knowledge we might have about bone physiology and structure. We need to realize that the question stem is trying to have you call to mind whatever you know about bone remodeling and the factors influencing bone density.
What do we know about bone remodeling? We know that new bone can be deposited onto existing bones by osteoblasts, and old bone can be removed by osteoclasts. When large forces are applied to bones, such as the forces experienced by bones due to the weight of holding up the body, osteoblasts are activated and they deposit new bone to withstand these forces.
The animal with the strongest and heaviest bones is the one whose bones are probably experiencing the greatest strain and stress due to bodyweight. That’s exactly the case for the land-dwelling mammal. Let’s see if we have any other interesting options.
A water-dwelling mammal would experience a buoyant force that would reduce the mammal’s apparent bodyweight. The bones would not be experiencing the greatest strain and stress. Answer choice A remains the best option.
Birds need to be able to fly, so they are unlikely to have a large bodyweight or heavy bones. Answer choice A is still our superior answer.
. Just like answer choice B, the amphibian would experience a buoyant force that would reduce its apparent bodyweight. We’re left with our best answer choice, answer choice A: A land-dwelling mammal.
This question is testing primarily your ability to remember relevant content information. Let’s go over some high-level details regarding the ectoderm, mesoderm, and endoderm. We can use a visual to help demonstrate as well.

As you can see from the visual, and you should know from your content, the ectoderm includes the central and peripheral nervous systems as well as the epidermis of the skin. The mesoderm includes the body’s connective tissues (such as bone, adipose, cartilage, and blood), the muscles of the body, the circulatory system and all its vessels (the heart, veins, and arterioles), the reproductive system, and the urinary system. The endoderm includes the GI tract and specialized digestive organs.
Based on this high-yield list, the heart and blood vessels both differentiate from the mesoderm, so only Answer A: II only is correct.

This question relies on your understanding of eukaryotic and prokaryotic cell structure. Oxidative phosphorylation is the process by which ATP is formed due to the transfer of electrons through the Electron Transport Chain (ETC).

As a brief reminder, during the eukaryotic ETC, electrons are brought by electron carriers such as NADH to the protein complexes embedded in the mitochondrial inner membrane. These electrons are passed from complex to complex until they are finally donated to oxygen. During this process, protons are pumped from the mitochondrial matrix into the mitochondrial intermembrane space. The buildup of H+ within the intermembrane space creates an electrochemical gradient of H+ ions. ATP Synthase then allows these H+ ions to move down their gradient, which releases energy. ATP synthase harnesses this energy to form ATP! This process is very similar in prokaryotes, with a slight difference – prokaryotes don’t contain mitochondria, so where would they have an electron transport chain capable of powering oxidative phosphorylation?

Bacteria can have a cell wall, however, this structure would be on the very outside of the bacteria, and not conducive to ATP production.
Bacteria, like eukaryotes, have ribosomes. However, these ribosomes are small and mobile, and as with eukaryotes are simply used to produce proteins.
This answer can immediately be eliminated, because bacteria do not have a nuclear membrane.
This is correct. Bacteria contain their electron transport chains within their plasma membrane. As a reminder, structurally, bacteria contain no membrane bound organelles. In a nutshell, they are just a big plasma membrane encapsulating cytoplasm, genetic material, ribosomes, and some miscellaneous materials. It is very efficient for bacteria to have their electron transport chain on their inner plasma membrane, because this means that ATP production can be coupled to processes that are located on the membrane which influence motility – for instance, ATP produced on the plasma membrane can be immediately used to power rotation of the bacterial flagellum.
Here we need to pay careful attention to each of the functions mentioned in the question stem. We have to make sure that whichever answer choice we pick has all of the listed functions, as opposed to just one or two of them.
The liver is primarily involved in metabolism, toxin detoxification, and the production of several compounds, such as bile, cholesterol, and various important blood proteins. Its primary functions are not those listed in the question stem, so we can eliminate it.
For the purposes of the MCAT, you should associate the spleen with two major functions. Firstly, the spleen is a lymph organ, and it has the function of housing immune cells which filter the blood for pathogens. In short, its first function is immune surveillance. Second, the spleen is associated with storing blood and recycling old red blood cells. We can eliminate the spleen.
The kidney is primarily associated with the production of urine, but urine production can be regulated in several important ways. For one, if the body detects changes in pH, the kidney can reabsorb more or less hydrogen ions from the urine back into the blood. Acid-base balance, check. The kidney can also reabsorb more or less water from the urine back into the blood, which will increase or decrease blood volume, which in turn will increase or decrease blood pressure. What about the removal of nitrogen wastes? Urea, a byproduct of protein catabolism, is filtered out of the blood and into the urine by the kidney.
The large intestine is mostly involved in the absorption of nutrients and the excretion of fecal waste, so we can eliminate this answer choice. Answer choice C was the only one of our options that hits all of our criteria, so we’re sticking with Option C as our best answer choice.
Full disclosure here, this is a slightly outdated question. The modern MCAT is not very likely to test you on extremely niche details of cell biology, but we still want to make sure we can answer this question in our practice. What this question is implying is, when we look closely at the Golgi apparatus, we can see that it has many folds in its membrane. Which other organelle would look similar to this?
The nucleolus is a membraneless structure sitting within the nucleus, responsible for ribosome production. No membrane, so we most likely will end up eliminating this answer choice.
Mitochondria contain two membranes with a number of folds, so we can hold onto this answer.
The plasma membrane is… well, a membrane. But this is a membrane without folds. Recall that was one of the big qualities we mentioned about the Golgi apparatus, so we can eliminate this answer choice. We’re still liking answer choice B as our best answer so far.
Smooth endoplasmic reticulum contains a membrane, and this membrane contains an extremely high number of folds, very similar to the Golgi apparatus. Therefore, this structure is more similar visually to the Golgi than the mitochondria.

A compound that inhibits monoamine oxidase (MAO) should have which of the following effects on NE concentration? We’ll want to revisit Figure 1 from the passage and what the author mentions about MAO and the effect on norepinephrine concentration.

Figure 1 is in the top of the image above. We also have an excerpt from the passage right below the image. It says “After its release, NE in the synaptic cleft is inactivated primarily by active transport back into the nerve terminal (Figure 1), where it is either broken down by monoamine oxidase (MAO) or sequestered into vesicles.”
The function of MAO is to break down norepinephrine, which we see near the top left of Figure 1. What would happen if MAO was inhibited? Norepinephrine would take the only alternative route here, and would be sequestered into vesicles, where it’s stored. And we see the arrow pointing here to the sequestered norepinephrine. MAO is an enzyme that essentially cleans up neurotransmitters like norepinephrine. So, what does the absence of MAO do to norepinephrine concentration? The concentration of norepinephrine in the neuron is increased.
One thing I want you to note. The author provides a beautiful figure that makes visualizing much easier. We can’t always rely 100% on figures like these to answer our questions. But when they support what the author mentions in the passage, it’s easy to see what would happen in specific situations like inhibiting one of the two potential pathways. If we cut out one of the only two options for norepinephrine in our figure, we’re expecting the remaining option will have a surplus of norepinephrine.

This answer choice is addressing norepinephrine concentration outside of the neuron. MAO is present and active inside the neuron, and that’s where we expect norepinephrine concentration to vary. I want to hold on to this answer choice for now. It is possible that if MAO is not breaking down norepinephrine, we may also see additional norepinephrine in circulation near the junction in general.

This answer choice is more closely related to our breakdown of the question, what we broke down in the Figure, and what we read in the passage. MAO breaks down norepinephrine within the neuron. If we don’t have MAO activity, we have excess norepinephrine in the neuron (at least compared to before). This is a superior answer to answer choice A, because it’s directly addressing the question being asked.

This answer choice is the opposite of our breakdown, and what we expect to happen with intraneuronal norepinephrine concentration. MAO breaks down norepinephrine, so inhibiting MAO will cause an increase in norepinephrine concentration, not a decrease. Answer choice C contradicts information from the passage and my breakdown of the question.

This answer is similar to option A where it addresses extraneuronal norepinephrine concentration instead of intraneuronal. We said MAO is present within the neuron, and also breaks down norepinephrine. That means the norepinephrine concentration inside the neuron would increase. We don’t expect a decrease in extraneuronal norepinephrine concentration. That means we’re left with our correct answer, answer choice B: Increase the intraneuronal NE concentration.

Which of the following processes is LEAST directly influenced by adrenergic drugs? Key word here is LEAST. We expect 3 answers to be influenced by adrenergic drugs to some extent. And one to be least directly influenced. We know adrenergic drugs mimic activation of the sympathetic nervous system-so we’ll use our knowledge of the sympathetic nervous system. The sympathetic nervous system controls the body’s automatic response to danger. That includes increasing the heart rate, dilating the blood vessels (or constricting in some situations), slowing digestion, and moving blood flow to the heart, muscles, and brain. We want an answer choice that would be least affected by activation of the sympathetic nervous system. So least related to these responses.

Peristalsis involves contraction of smooth muscle, and helps the transport of food along the digestive system. I mentioned in my breakdown of the question, adrenergic drugs will affect the digestive system, and smooth muscle. Why is that? Because the drugs mimic activation of the sympathetic nervous system.
This answer choice is similar to answer choice A. I mentioned adrenergic drugs will affect the digestive system. Secretion of digestive enzymes is a process that’s part of the digestive system. Adrenergic drugs will directly influence the secretion of digestive enzymes as well. No clear cut winner between A and B, so let’s keep considering our additional options.

This answer choice is a little tricky. Adrenergic drugs will directly influence the secretion of digestive enzymes and peristalsis-we covered that already. As a result of the decreased secretion of digestive enzymes, we might have less enzymes released overall and less enzymatic activity as a whole. But the enzymes that do get secreted, those aren’t going to be affected directly. The sympathetic nervous system is not going to affect the activity of individual enzymes, but rather the secretion of enzymes and the digestive system itself will be more directly affected. Note the difference and the distinction. A and B are directly influenced by adrenergic drugs and sympathetic nervous system control. Even though fewer enzymes are secreted, enzyme activity won’t be affected directly. Answer choice C is now the superior answer choice.

I mentioned in the breakdown of the question that blood flow to muscles and organs is influenced by sympathetic nervous system activation. I specifically talked about typical beta responses including increased blood glucose concentration and dilation of blood vessels supplying deep muscles and internal organs. Nutrient delivery would be affected directly by adrenergic drugs. The best answer here is going to be answer choice C. The enzymatic breakdown of food molecules would not be directly affected the same way as the other answer choices.

Applying a drug which blocks the absorption of NE into the adrenergic nerve terminal will result in. I’m going to jump back to that beautiful figure from the passage one more time. We can answer this question by identifying the effect of blocking the absorption of norepinephrine.

We have Figure 1 from the passage above. And we can quickly go through what the author talks about in regards to norepinephrine absorption. The author says “After its release, NE in the synaptic cleft is inactivated primarily by active transport back into the nerve terminal (Figure 1), where it is either broken down by monoamine oxidase (MAO) or sequestered into vesicles.” We just talked about this in a previous question.
Which part of our figure is being blocked? We’re blocking the absorption of norepinephrine into the nerve terminal. You can print and put an ‘X’ through it if that makes it clearer. Visualizing always helps if something is unclear. Now we have an increase of norepinephrine in the synaptic cleft that’s not being absorbed and entering back into the nerve terminal.
That surplus of norepinephrine can mean two things:
First, there can be an increase of COMT activity, which means more breakdown of norepinephrine. Second, we have active norepinephrine interacting with the adrenergic receptors. So, our answer is going to revolve around one of these two outcomes.

This answer choice is consistent with one of our predictions. We mentioned a surplus of norepinephrine in the synaptic cleft means that excess norepinephrine is able to continue to interact with adrenergic receptors. Stimulating adrenergic receptors means activation of the sympathetic nervous system. Answer choice A is consistent with my above breakdown of the question.

This answer choice directly contradicts our predicted answer. Without the usual uptake of norepinephrine, we have a surplus that can stimulate adrenergic receptors. That stimulation increases sympathetic responses, not decreases. Answer choice A is superior.

This answer choice is not going to be as relevant as answer choices A and B. MAO is found inside the nerve terminal, but the norepinephrine is no longer making it back into the nerve terminal. Just by looking at the Figure and noting what’s going on, we can eliminate this answer choice for not being relevant to the situation presented in the question stem.

This answer choice is the opposite of my breakdown and what the author talks about in the passage. We expect that a surplus of norepinephrine will mean we can have an increase in COMT activity, not decreased destruction. That means we’re left with our correct answer, answer choice A: increased sympathetic response.
The amount of NE released by sympathetic nerve terminals will be most strongly influenced by a change in which of the following? This answer is open-ended and very broad. The author could be talking about something in the passage, or about norepinephrine activity in general. Because the question is so open-ended, we are going to do things a little differently here before we really dive into the question. We’re going to glance at our answer choices to get a better sense of what the test-maker wants for an answer.
We have alpha receptor sensitivity and density, concentration of extracellular calcium ion, and COMT activity. The only one of those mentioned in the passage was COMT, which breaks down any norepinephrine not transported back into the nerve terminal.
Right away, we’d be inclined to not pick answer choice D. COMT activity would influence the amount of norepinephrine absorbed into the adrenergic nerve terminal. Not necessarily the amount of norepinephrine released by nerve terminals. Alpha receptors are also located outside of the nerve terminal. If we have a change in sensitivity or density, the can affect alpha response, but won’t affect the norepinephrine released by the nerve terminal.
Lastly, we have concentration of extracellular calcium ion. This is where our general knowledge comes into play. We’re told we have a change in extracellular calcium ions. If we reach the critical threshold, the neuron will fire an action potential. Calcium channels are opened and calcium flows into the nerve terminal. If you’re looking at the figure: calcium movement from the middle of Figure 1, to the left side, the nerve terminal (Once again, pull up the figure if that helps you visualize better). The vesicles that are storing norepinephrine will release their contents directly into the synaptic cleft by exocytosis. Ultimately, changing extracellular calcium levels can directly influence whether we have an action potential and the ejection of norepinephrine into the synaptic cleft.
For this question, we actually went through all of our answer choices already. This question was a little different. The process I used to break it down was different than we’re used to. The question was so open-ended that I had to compare all of the answer choices at once before jumping into the passage or the science. We’re left with our correct answer, answer choice C: changes in extracellular calcium levels would most strongly influence the amount of norepinephrine released by sympathetic nerve terminals. We can eliminate answer choices A, B, and D because none of those had the same influence.

Based on the passage, which of the following conditions would most likely be aggravated by drugs that increase beta-adrenergic receptor activity? If we know the exact effects, we can tie those into the conditions listed in our answer choices. That means the answer is going to come from the passage initially. Let’s pull up the necessary excerpt and note the effect of increased beta-adrenergic receptor activity:

We want to note what the author says about beta responses. It says “Typical beta responses include increased blood glucose concentration and dilation of blood vessels supplying deep muscles and internal organs.” We expect one of our conditions in the answer choices will be exacerbated by these symptoms.

Remember our prediction is increased blood glucose concentration and dilation of blood vessels. The common cold is a viral infection, usually of the upper respiratory tract. This condition wouldn’t be affected directly by beta responses.

This answer choice looks promising. Diabetes mellitus is a disease where the body doesn’t produce enough insulin, or the insulin response is abnormal. As a result, blood glucose levels become abnormally high. If we have a typical beta response that causes increased blood glucose concentration, this condition can get much worse. The body can’t respond properly to this increased blood glucose concentration. This sounds like a better option than answer choice A because answer choice A wouldn’t be directly affected by the typical beta responses.
The author is trying to tie this condition into the change in pupil size. We can note that typical alpha-adrenergic response includes widening of the pupil, but that would actually improve night vision, and vision in low light. This answer choice is not answering the specific question being asked, and it’s related to alpha-adrenergic receptor activity-not beta. Answer choice C is not a good option here.
Lactic acid will accumulate when there is high energy demand in muscles, and there isn’t enough oxygen. Some of you that work out or lift weights will be familiar with this in your day-to-day lives. But a typical beta response includes dilation of blood vessels supplying muscles. What does dilation mean? The blood vessels widen and blood can reach the muscles more easily and quickly. We’re expecting the opposite effect than what answer choice D mentions, and this answer choice contradicts information from the passage. We can pick our correct answer, answer choice B: Diabetes mellitus.
If a cell’s membrane potential changes from –60mV to –70mV after treatment with an adrenergic drug, the NE receptor is most likely linked to. What information do we have going into this question? We know the drug stimulates adrenergic receptors, and we know there’s a change in membrane potential from -60 millivolts to -70 millivolts. A visual will help us really understand the question and how we should answer this.
Here’s a visual straight from our website. We’re going over the significant events in the formation of an action potential. We want to keep in mind the change the question stem mentioned: From -60 millivolts to -70 millivolts. That happens during repolarization, and right before hyperpolarization. If you need to, you can mark it on the visual. What’s happening during repolarization? Potassium channels open and potassium begins to leave the cell. At the same time, Na+ channels close. Eventually, the membrane becomes hyperpolarized as K+ ions continue to leave the cell. We’re not interested in hyperpolarization. We’re sticking to the range from -60 millivolts to -70 millivolts where we have the opening of potassium channels to release potassium from the cell. That eventually restores the cell membrane to its resting potential of -70 millivolts after an action potential, and we can see all of that in the visual.
G-proteins are transmembrane proteins. They act as on and off molecular switches by binding to either GDP or GTP. Essentially, they’re secondary messengers. We can hold on to this answer for now, but it’s not directly relevant to the question stem and action potentials. The only reason we’re keeping it is in case we don’t find something better.
This is similar to answer choice A. Adenylate cyclase produces the secondary messenger: cyclic AMP. The enzyme is activated by G-protein-coupled receptors. However, this is again not directly relevant to the very specific information given in the question stem. We’re not focused on these secondary messengers.
This answer choice is more relevant to our breakdown, visual, and the question stem. The only issue here is, voltage gated sodium channels open and the membrane depolarizes. That’s not what we’re focused on. We’re more concerned with the change from -60 to -70 millivolts which is repolarization. Still, answer choice C is the most relevant answer so far, and directly ties to our breakdown and the question stem.

Here we go, this is what I was looking for as I broke down the question. I said at peak action potential potassium channels open and potassium begins to leave the cell. That continues to happen through a membrane potential of -60 millivolts to -70 millivolts. That’s exactly what we were looking for in the question stem, so we’re left with our correct answer, answer choice D: a potassium channel.

The osmotic concentration of plasma proteins in the venous side of capillaries helps reduce the amount of interstitial fluid in tissues by inducing. This is almost a standalone question that can be answered without going back to the passage. I talk about this a lot where the test-maker will ask a question that’s tangentially related to the passage, but we don’t need the information in the passage to actually answer. We’re going to focus on plasma proteins and we’re explaining the movement of fluid from interstitial fluid back into capillaries, or osmotic pressure. That pressure is the result of concentration gradients. We have plasma proteins that can’t move across the semipermeable capillary cell membrane-meaning they stay in the plasma. Blood is going to have a higher plasma protein concentration and lower water concentration than interstitial fluid. How can that be fixed? Two ways: First way is protein diffuses across the membrane and out of the capillary, but we said proteins are too big and that can’t happen. The other way is through the diffusion of water. That higher protein concentration in the capillaries attracts water, so water is drawn from the interstitial fluid back into the capillary.
This answer choice matches our breakdown of the question, and greatly simplifies it as well. We have a higher concentration of plasma proteins, so we have passive water diffusion along a concentration gradient. That reduces the amount of interstitial fluid in tissues. Decent answer choice here that answers the question being asked and it’s consistent with my breakdown of the question.

This answer choice is not related to the question stem or my breakdown of the question. We’re dealing predominantly with plasma protein concentration and diffusion of water. We’re not focused on passive ion diffusion along an electrochemical gradient. Rather we want an answer that focuses on water, like answer choice A.

This is similar to answer choice B. We’re not focused on ion transport as much as we are on the movement of water and passive water diffusion. Answer choice C is inferior to answer choice A.
This answer choice insinuates we’re going to see active transport in the movement of water. That’s not the case. I said in my breakdown that there’s water drawn from the interstitial fluid, into the capillary. That’s done passively; there’s diffusion along a concentration gradient. This answer choice contradicts my breakdown and what we should know from our general knowledge. That means we’re left with our correct answer, answer choice A: passive H2O diffusion along a concentration gradient.
Capillaries in the kidney and elsewhere in the body maintain fluid homeostasis by balancing hydrostatic and osmotic pressures. Which of the following is the initial effect of a blood clot forming on the venous side of a capillary bed? At first glance, the way we attack this question and the knowledge we’ll be using is very similar to the previous question. Just like before, we’ll use our knowledge of osmotic and hydrostatic pressure, and I’ll explain what happens following blood clot formation. First thing we want to note is how blood flows. We have blood flowing from arteries, to capillaries, and to veins. That means we can still have blood flow from arteries to capillaries. But there’s a blood clot further down, meaning we’ll have a buildup in the capillaries. Extra blood in the capillaries means increased hydrostatic pressure. That’s the pressure exerted by blood against the wall of the capillaries. Now this is when we tie into our previous question and our previous breakdown. How can we fix the buildup of blood and increased pressure? Through the diffusion of water into the interstitial fluid. Previously, we focused on osmotic pressure mostly as there were fluids drawn from the interstitial fluid back into the capillary. Now we’re seeing the opposite because of the buildup.
This is similar to what I just mentioned in the breakdown of the question: there’s blood buildup in the capillaries. Increased hydrostatic pressure. How do we compensate? Through a new flow of fluid out of the capillaries. That’s exactly what this answer choice is presenting to us. Our initial effect, is we have fluid flow into the interstitial spaces to account for the increased hydrostatic pressure.
This is the opposite of what I alluded to in my breakdown. We have an increase of hydrostatic pressure which would cause an increase in net fluid flow, not a decrease. This answer choice is unreasonable.

Changes in osmotic pressure are expected eventually if we have a blood clot. But will that be the first effect? Capillary hydrostatic pressure is the force that drives the initial move. That movement of fluid out of capillaries, and into the tissues. Capillary osmotic pressure can play a role down the line, but for the time being, we’re going to stick with the initial effect mentioned in the question stem. Answer choice C is inferior to answer choice A.

This is similar to answer choice C. We’re asked about the initial effect of a blood clot forming on the venous side of a capillary bed. In my breakdown and as we’ve gone through our other 3 options, we noted the initial effect is a movement of fluid from capillaries to interstitial spaces to account for the increased hydrostatic pressure. Best option here after considering each one is answer choice A: Net fluid flow in the direction of interstitial spaces will increase.
Which of the following changes in flow rate or in solute concentrations would NOT occur if the blood inflow rate were increased, increasing the pressure in the dialysis chamber? Verbiage here is key, and we want to be careful. If we have increased blood inflow rate, that increases the pressure in the dialysis chamber. That means there is now more blood and increased pressure. We want to know which of the 4 changes listed as answer choices would NOT occur. We’re going to focus on the transport of blood through the hemodialysis units. That means we’ll focus on flow rate, solute concentrations, and blood pressure in general.
Increased inflow rate means increased pressure in the dialysis chamber. First and foremost, we know increased inflow rate would like also increase the volume of blood traveling through the unit and exiting back to the patient. Next, we expect that increased flow also means the dialysis or filtration rate would also increase. More volume coming in and higher pressure means more filtration taking place. In terms of solute concentrations, we’d still expect diffusion to take place across the semipermeable membrane. As we get into specific situations and attack our 4 answer choices, we have to remember we want an option that will NOT occur. Never let the fact that you misread a question be the reason you get the question wrong.
This first answer choice contradicts what we’re looking for. Why? Because we want an option that will not occur. That means answer choice A is a true statement, but an incorrect answer choice. We have greater blood volume inflow, which also means increased blood outflow if we have a properly functions hemodialysis unit.
This answer choice is assuming proteins can cross the semipermeable membrane, which is untrue. The author says “Protein molecules are too large to diffuse through the membrane” in the passage itself. That means we’re not expecting osmotic concentration of proteins to change. Because we’re expecting answer choice B will not occur, that makes it a good answer choice.
This answer choice ties into what I just said previously. We might not have an increase in osmotic concentration of proteins in the blood outflow, but this answer choice tells us that concentration may remain unchanged. That’s exactly what we expect to happen, so we can eliminate answer choice C.
This is an answer choice that I explicitly mentioned in my initial breakdown of the question. There is more volume coming in and higher pressure, so that means more filtration is taking place. The sheer volume and pressure are causing the increase in filtration rate. That means this answer choice goes against what we wanted in our question stem. We’re left with our correct answer, answer choice B: The osmotic concentration of proteins in the dialysate fluid would increase.
Bicarbonate ions in the blood and the dialysate are important for maintaining physiological levels of. Let’s go through what we know about bicarbonate ions; they’re important for maintaining physiological levels of one of our 4 answer choices. Think back to your biochemistry lectures and maybe even your MCAT content review. You probably know bicarbonate ion from pH control in the body. When we have acidosis, tubular cells with reabsorb more bicarbonate from tubular fluid. Collecting ducts secrete more hydrogen and generate more bicarbonate. When we have alkalosis, kidneys may excrete more bicarbonate by decreasing hydrogen ion secretion. That’s the big relationship we’re aware of when it comes to bicarbonate, so we can see if any of our answer choices are consistent with this role.
We didn’t have much exposure to bicarbonate in the passage beside what we say in the table of solute concentrations. That means we’re relying heavily on our general knowledge and what I said in my breakdown of the question. I didn’t mention anything about bicarbonate ions maintaining water levels, but we’ll still hold on to answer choice A until we find something superior.
This answer choice is similar to option A. We don’t have any mention of chloride in our breakdown of bicarbonate. Another thing we want to note, when we’re going through the excretory system and homeostasis, we focus on water levels quite a bit. Even if that doesn’t mean in relation to bicarbonate. Chloride, on the other hand, isn’t as prominent in maintaining homeostasis. That means we’re going to stick with answer choice A as our superior answer choice for now, just because answer choice A is a broader answer.

This answer choice matches part of my breakdown exactly, and also gives us some more confidence in eliminating answer choices A and B. We know the kidneys will use bicarbonate as a tool in controlling pH in the body. And what does that mean specifically? Bicarbonate ions are important for maintaining physiological levels of hydrogen ions. We’re liking this answer choice, but we still want to take a look at answer choice D to be thorough.

This answer choice is similar to answer choice B. It’s not as broad of an answer choice as water, and it doesn’t match what I said in my breakdown, or what we know from our general knowledge. Think of this from a logical and critical thinking perspective. Bicarbonate ions shouldn’t affect glucose concentration or the physiological levels of glucose. Let’s stick with our superior answer, answer choice C: hydrogen ions.

Why are high concentrations of sodium included in the dialysate (Table 1)? First thing I’ll want to do here is actually pull up Table 1. Once we look at the table, then we can start looking for reasoning behind the high concentrations of sodium.

We have Table 1 above and we’re going to focus on sodium specifically. We want to know why there are relatively high concentrations of sodium included in the dialysate. We have a range that’s similar to what we see in individuals with and without renal failure, but what might be the reason for that? One way we can attack this is we can consider what the effect would be if we had a much lower concentration of sodium in the dialysate. We know water can diffuse across the dialysis membrane and it will diffuse when we have a significant concentration difference. The dialysate has to have similar sodium concentration to the patient’s blood, or we might see an unwanted shift in fluid and pressure.

This answer choice is the opposite of what I just went over. We’re not trying to induce water movement from the blood into the dialysate fluid. If that were the case, we’d need sodium concentration in the dialysate fluid to be higher than that of the patient’s blood. We have nearly identical concentrations right now, so we’re not expecting net water movement.

This answer choice is similar to answer choice A. If we had a high enough sodium concentration, we could induce water movement from the blood, into the dialysate fluid. If we have water movement into the dialysate fluid, that could mean high osmotic pressure. But again, we have nearly identical concentrations, so we’re not expecting net water movement.

What does isotonicity mean? Essentially, we’re saying the concentration of sodium in the blood and the dialysate are the same. Said differently, there’s no net movement of water in or out of the membrane. I’m liking answer choice C. Answer choices A and B both implied a net movement of water which contradicts what I saw in Table 1 and in my breakdown.

This answer choice is not relevant to sodium levels. Sodium concentration is not going to compensate for levels of urea nitrogen and creatinine in the blood. We’re trying to get those waste products out of the blood. But sodium concentration is not going to affect the levels of urea nitrogen and creatinine. Answer choice D isn’t great, so we’re left with the only answer choice that is consistent with my breakdown of the question, answer choice C: To maintain isotonicity of the dialysate solution with blood.

Which of the following figures (A–D) shows expected solute filtration rates (mEq/mL-min) as a function of molecular weight for two dialysis membranes: Membrane 1 with large pores and Membrane 2 with small pores? We’re going to approach this a little differently. Normally I try and break down the question before diving into the 4 options. This time we want to look at the 4 given figures. One figure will correctly show the graph of filtration rate compared to solute molecular weight for two membranes.

We also want to recall some details from the passage. Dialysis membranes are semipermeable-they allow diffusion of solutes with molecular weights up to 1000–2000 Daltons, depending on the size of the membrane pores. In general, we expect filtration rate to decrease as molecular weight increases. Above a weight of 2000 Daltons, we expect filtration rate to be essentially zero in both membranes. In the case of this specific question (which is what we care about right now), membrane 1 has large pores, membrane 2 has small pores. That means heavier solutes can pass through membrane 1 than can pass through membrane 2. More molecules can pass through membrane 1 at every solute molecular weight.

Now, we can compare all 4 graphs at once. In general, we expect filtration rate to decrease as molecular weight increases. Why is that? Because as solute molecular weight increases, fewer molecules will pass through the membrane. Right away we can eliminate answer choices A and B. Those both show an increase in filtration rate as we see an increase in solute molecular weight. Next thing we want to note, is we said membrane 1 will have a higher filtration rate when we have larger solute molecular weights. Why is that? Because membrane 1 has larger pores. Let’s look at answer choices C and D. Answer choice D shows a greater filtration rate at higher solute molecular weight for membrane 1. That’s consistent with our reasoning, so we can stick with the best option here: answer choice D.

Which of the following pieces of experimental evidence best supports the Mosaic Hypothesis? We can quickly pull up an excerpt from the passage and review the main points from the Mosaic hypothesis. Remember, I bring these parts of the passage back up for the sake of demonstration. I want you to be able to follow along with my thought-process. This is not something you absolutely need to do for every question. For science passage, revisit the passage only when you need to (typically that’s for small details or specific numerical values).

We have the part of our passage covering the mosaic hypothesis above. Again, it’s just so you can read along. I don’t advocate going back and re-reading the passage multiple times when you’re trying to answer questions.
The author really does their best to make this passage as structured and straightforward as possible. Last sentence gives us a summary of the hypothesis and conclusion. It says the “fate of developing cells is determined by the cells’ unequal content of determinants, and that cell lineage is unaffected by external conditions or by the position of a cell in the embryo.” And the experiment this biologist conducts shows exactly that. The scientist kills one cell of a two-celled embryo. The surviving cell forms only half an embryo. So, we’re looking for an answer that shows something similar. We want to find an example of the fate of a cell depending on intrinsic characteristics, not on position in the embryo.

This answer choice implies a single egg has plans for, and produces two or three individuals. The fact that the egg can develop into a complete organism supports the regulative hypothesis, not the mosaic hypothesis.
This answer choice is implying that cells contained identical information to produce identical results. The fact that two normal-sized heads are produced shows that each cell didn’t have distinct substances needed for its own development. Each likely had a complete set of determinants, and two heads are produced by accident, like the question stem mentions. Again, another situation that’s consistent with the regulative hypothesis.

The “partial embryos” mentioned in this answer choice is a good start. We’re told separated cells continue to divide and produce partial embryos. That sounds like what happened in biologist 1’s experiment and supports the biologist’s hypothesis. We can keep this answer choice as a prime example of experimental evidence supporting the Mosaic hypothesis. This automatically becomes our best answer choice because A and B support the regulative hypothesis.

This answer choice is essentially just giving us the definition of the regulative hypothesis. Biologist 2 concluded that each cell is capable of developing into a complete organism and contains the same genetic information. That means we’re left with our best answer, answer choice C: Separated cells of two-celled embryos continue to divide, producing partial embryos.

The nucleus of a frog egg is destroyed by radiation and replaced by a nucleus from a differentiated gut cell of a tadpole. The resulting egg is activated and develops into an adult frog. Are the results of this experiment more consistent with the Regulative Hypothesis or the Mosaic Hypothesis? We’re going to compare the conclusions of the two biologists. Like I mentioned before the first question here, the big picture is often enough to get you close to the correct answer.
Mosaic hypothesis says cell fate is predetermined and is based on intrinsic characteristics. Regulative hypothesis says cell fate is dependent on environmental factors, like interactions with other cells. Only in the regulative hypothesis do we suggest cells have a complete set of determinants and can develop into complete organisms. That’s exactly what’s happening here. Even though the nucleus of the egg cell is replaced by the nucleus of a gut cell, we still get a complete, adult frog. How is this possible? If the regulative hypothesis is correct.
First part of our answer choice is consistent with our breakdown of the question. The results are consistent with the regulative hypothesis. But the reasoning here is a bit iffy. The radiation is what destroyed the nucleus of the frog egg. That wasn’t necessarily what caused the fertilized egg to develop into a frog. In fact, it might’ve even prevented that from happening. We like the regulative hypothesis part, but not so much the reasoning here.

This answer choice also starts out consistent with our breakdown. We’re looking for an answer with regulative hypothesis. Is the reasoning here better than answer choice A? This reasoning is almost verbatim what biologist 2 concludes after conducting the experiment with sea urchin embryos. Answer choice B is a superior answer that’s supported by the correct (regulative) hypothesis according to our breakdown.

This answer choice contradicts what I said in by breakdown of the question. We want an answer that mentions regulative hypothesis. This answer assumes something that we cannot assume is true. We don’t know if there were genes that were retained in the nucleus of the frog cell after radiation treatment. We can eliminate answer choice C as well.

This answer choice also contradicts our breakdown of the question. We also don’t know about genes from another cell being added. We want to stick with an answer choice that supports the author’s conclusion and mentions the mosaic hypothesis. That means we’re left with our correct answer, answer choice B.

The validity of the Regulative Hypothesis could best be demonstrated in an organism by showing that. This is going to be similar to our previous question where we’re focused on the big picture from the passage. Specifically, we’re going to focus on the regulative hypothesis and find an answer choice that’s consistent with the hypothesis. Regulative hypothesis says cell fate is dependent on environmental factors, like interactions with other cells, and their position in the embryo. Cells have a complete set of determinants and can develop into complete organisms.

This answer choice contradicts what I’ve said in the breakdown of the past few questions, and what the author mentions in the passage. Even if cells are separated, they will still develop into a complete, albeit smaller embryo. The “incomplete” goes against the passage and breakdown.

This answer choice is describing the mosaic hypothesis. In the mosaic hypothesis, the author mentions the cell contains substances needed for its own development. In the regulative hypothesis, the author discusses the fate of cells depending mainly on environmental factors. We’re still comparing answers A and B to the rest of the answers. We haven’t been able to rule either one out.

This answer choice is consistent with the breakdown of the question and biologist 2’s experiment. Biologist 2 concluded the fate of developing cells depends mainly on environmental factors and on their position in the embryo. So even if a cell is added to a new environment, transplanted embryonic cells show position-dependent development. Cell fate isn’t predetermined, which means we can now eliminate the inferior answers: A and B.

This answer choice is also consistent with the mosaic hypothesis. Genes determining cell development are distributed asymmetrically, as needed. Looking at the regulative hypothesis instead, we’re told each cell contains a complete set of determinants. This answer choice also contradicts the passage and the breakdown of the question. We’re left with the correct answer, answer choice C: transplanted embryonic cells show position-dependent development.

Which of the two hypotheses in the passage most closely fits the present-day understanding of human differentiation? How are we attacking this question? We’re going to relate the mosaic or regulative hypothesis to present-day understanding of human differentiation. In humans, cells have the capacity to mature into different cell types. Organs develop from the germ layers through the process by which a less-specialized cell becomes a more specialized cell type. During cell differentiation, embryonic stem cells express specific sets of genes that determine their ultimate cell type. That is more consistent with the regulative hypothesis, and cell fate isn’t predetermined.

I’m not a fan of the first part of the answer choice. I said we’re looking for an answer that says regulative hypothesis. Let’s be thorough though. How do we feel about the reasoning here? Germ cells will differentiate and undergo meiosis to produce a haploid set of chromosomes. But ultimately, haploid gametes unite to form a diploid zygote, that develops into a new individual.
We don’t like the first part of the answer choice. We said we’re looking for an answer choice that talks about the regulative hypothesis. Additionally, activation of genes at different times does not support the Mosaic hypothesis. Reasoning here isn’t great, and the first part of the answer choice contradicts what I said we’re looking for in the breakdown.
First part of this answer choice we’re liking, but let’s get into the reasoning. Each embryonic cell does receive a complete set of genes. But we’re told cell position is unrelated to differentiation, and we know that’s not true. A cell’s position influences the differentiation of cells all around it. That being said, this is still the best answer because the first part of the answer matches our breakdown of the question, and it’s factually correct.

This answer choice harnesses the correct parts of answer choice C, while also pointing out that cell position helps determine differentiation. What does that mean? We can eliminate answer choice C for implying cell position is unrelated to differentiation. We’re left with our correct answer, answer choice D: The Regulative Hypothesis, because each embryonic cell receives a complete set of genes, and cell position helps to determine differentiation.

The cell nucleus below contains the chromosomes of a sea urchin embryo at the two-cell stage.

Which of the diagrams below best represents the nucleus of an embryo at the 64-cell stage grown from this cell?

We’re going to focus on sea urchin embryos, and the regulative hypothesis. The author mentions in the passage that even when the two cells of a sea urchin embryo separated, each cell developed into a complete embryo. Each cell retained a complete set of determinants. What does that mean? All cells are going to have the same genetic makeup. We’re expecting the nucleus of an embryo at the 64-cell stage to look the same as the cell nucleus at the two-cell stage. Why is that? Again, because when we look at the regulative hypothesis-all cells contain a complete set of determinants, and each cell is capable of developing into a complete organism.

This is going to be a straightforward choice here. We’re just looking for an answer choice that matches the nucleus in the question stem. Only answer that matches our criteria is answer choice D. What’s wrong with the other answers though? Why are we able to eliminate the other options? We have fewer chromosomes in answer choices A and B. We have extra chromosomes in answer choice C.

The validity of the Mosaic Hypothesis could best be demonstrated in an organism by showing that. This is almost identical to Question 35, only now we’re focused on the mosaic hypothesis. So how can we best demonstrate the mosaic hypothesis is valid? Mosaic hypothesis says cell fate is predetermined, and is based on intrinsic characteristics. Said more generally: the fate of developing cells is determined by the cells’ unequal content of determinants. Cell lineage is unaffected by external conditions or by the position of a cell in the embryo. Each cell contains only the substances needed for its own development.

This answer choice contradicts our breakdown of the question and what the author told us in the passage. Answer choice A is more describing the regulative hypothesis which says cells retain a complete set of determinants.

This answer choice is also more regulative hypothesis-centric. The mosaic hypothesis says cells of a developing embryo are independent and act as individual pieces of a mosaic. We’re not liking either one, but we’re still keeping answer choices A and B. We’re going to be aware that both contradict the passage, and the definition of the mosaic hypothesis. Ideally, we find a better option in answer choices C and D.

This is our third straight answer choice that is more relevant to the regulative hypothesis. The author mentions cell lineage is unaffected by external conditions. So far, all three answer choices have described evidence for the regulative hypothesis, not the mosaic hypothesis. Let’s see if answer choice D gives us a better option or else we’ll go back through the answer choices to find the best choice.

Bingo! We have an answer choice that’s consistent with the breakdown of the question and what we read in the passage. We’re told each cell contains only the substances needed for its own development, and the fate of the cell is determined by the cell’s unequal content of determinants. In other words, the cell’s fate is predetermined to an extent, and unaffected by the position of the cell in the embryo. That means we’re left with our correct answer, answer choice D: the fate of transplanted embryonic cells is independent of their new position in the embryo.

Radioactively labeled uracil is added to a culture of actively dividing mammalian cells. In which of the following cell structures will the uracil be incorporated? This is a very straightforward discrete. Basically, can you identify a part of the cell which contains uracil? What do we know about uracil? This is a nucleotide found only in RNA, so you should already be thinking about structures of the cell which contain RNA.

Chromosomes are a cell structure composed of proteins and DNA, not RNA, so we will likely eliminate this.

You probably know that ribosomes are used in translation, but you should also know that they are composed of ribosomes and a specialized form of RNA called rRNA. Therefore, uracil would likely be incorporated into a ribosome.
Lysosomes are an organelle whose structure has nothing to do with RNA. We’re still sticking with answer choice B as our best answer.

The nuclear membrane of a cell is composed of phospholipids and proteins, not ribonucleic acids.

A resident of a famine area who appears undernourished and extremely emaciated has eaten only starches for the past 3 months. A urine analysis shows that a large amount of nitrogen is being excreted. This is most likely evidence of:
This question is a little bit more complicated than some of the other standalone question contained in the biology question pack. There are several moving parts, so we’ll tackle the question stem step-by-step.
First, the question stem mentions an individual who is malnourished, lacking a consistent food source. This should immediately suggest that we will be dealing with metabolism content, and what should come to mind is the fact that the man is probably in a catabolic state. A catabolic state is one in which an individual has been fasting, and is low on immediately available sources of energy. An individual who is catabolic will need to break down parts of their body for energy.
Broadly speaking, if an individual has only been fasting for a few hours, their body will start producing hormones that promote catabolism, such as glucagon. Catabolic hormones will promote catabolic metabolic processes, such as glycogen breakdown, beta oxidation (fat breakdown), and protein catabolism.
So, the individual is catabolic. Next, the question stem mentions a great deal of nitrogen excretion through the urine. Why would nitrogen be in the urine? Well, most of the nitrogen in the urine is due to urea being filtered into the urine. Urea is a byproduct of protein catabolism. The gears should already be turning here. Let’s move onto the answer choices, and see if any of them reflect what we’ve discovered.
In a catabolic state, glycogen breakdown will become elevated. However, glycogen stores are finite and will only last a few days at most. If the individual has been starving for 3 months, it’s unlikely that he has an elevated rate of glycogen breakdown. Furthermore, the breakdown of glycogen is unrelated to nitrogen production, because simple carbohydrates do not contain nitrogen.
Bingo. Protein breakdown could definitely be the culprit for elevated nitrogen levels in the urine due to increased urea production, and we know that protein breakdown will be elevated due to the individual’s starvation.

Like answer choice A, the catabolism of fats would not explain the elevated nitrogen, because the storage form of fats (triglycerides) do not contain nitrogen.

The AAMC explanation suffices here, but to expand: urea is secreted into the urine with the intension of excreting it, not reabsorbing it. The normal function of the kidney is to not reabsorb nitrogenous products. There is no reason to believe that nitrogen being present in the urine is due to kidney failure. Answer choice B is going to be our best answer of the bunch.

A stable, differentiated cell that will NOT divide again during its lifetime would most likely be found in which of the following stages of the cell cycle? This question is testing your understanding of the various stages of the cell cycle. Recall that the cell cycle is divided into two major parts, interphase and mitosis. G1, S, and G2 are all normal parts of interphase. We can use a visual to help us get through this question and attack our four answer choices.

The G1, S, and G2 stages are all normal parts of interphase. The G1 stage is the very first stage of interphase, characterized by increased production of macronutrients and other cellular resources. However, being the first stage, G1 also functions as a sort of “checkpoint” for interphase. If the cell wants to continue growing and proceed toward mitotic division, it will move past G1. However, if the cell wants to remain dormant, for example, like how a mature neuron doesn’t want to actively divide, it can remain arrested in G1. In practice, what this means is that the cell is deactivating some factor that was initially responsible for promoting cell division. For example, the cell might self-destruct some of the DNA responsible for cell proliferation, or it might simply stop producing growth factors. Alternatively, a cell can be arrested in a different stage, called “G0”. The difference between G1 and G0 is that G1 arrest is usually reversible, while G0 arrest is usually not. This sounds like a good option for the time being.

If a cell is in the G2 stage, it is already well on its way toward mitotic division. Answer choice A remains the best option.

The letter “M” stands for mitosis. If a cell is undergoing mitosis, it’s actively dividing, and is clearly not under growth arrest! We can eliminate this answer choice.

Like G2, if a cell is in the S stage, it is already well on its way toward mitotic division. We can eliminate this answer choice. We’re left with our best answer, answer choice A: G1.
All of the following occur during normal inspiration of air in mammals EXCEPT:
For this question, we deep dive into respiratory physiology. Respiration can be broken down into two parts, inspiration and expiration. In general, during inspiration, the chest cavity needs to get larger to allow for air intake, while during expiration the chest cavity needs to shrink to expel air. You can visualize this by looking at the following figure:

Elevation of the ribcage would expand the chest, which is the goal of inspiration, so we can let go of this answer. Make sure to pay very close attention to the verbiage. The “EXCEPT” is key here.

The diaphragm is the skeletal muscle hanging out at the base of the lungs. When the diaphragm contracts, it shortens and flattens, pulling the lungs downward. By pulling the base of the lungs downward, the lungs will expand and allow for more air intake. Therefore, diaphragm contraction is associated with inspiration. However, the answer choice mentions diaphragm relaxation, so we like this answer.

How might pressure be related to inspiration, you ask? The pressure felt by the lungs is due to molecules of air colliding with the walls of the lungs. When pressure is greater in one area than another area, molecules of air will move from the area at higher pressure to the area at lower pressure. The goal of inspiration is to move air into the lungs. Therefore, if the pressure in the lungs is reduced relative to the area outside the body, air will move from outside of the body and into the lungs. Answer choice B is still the best answer here based on what the question stem is asking us.

During inspiration, the diaphragm and external intercostal muscles both contract to expand the rib cage. Therefore, we can eliminate this answer choice. We’re left with our best answer, answer choice B.

The antibiotic penicillin has the effect of inhibiting the production of the chemical peptidoglycan. Therefore, penicillin is likely to be most effective in treating infection by: Peptidoglycan should sound familiar. It’s a polysaccharide-protein molecule, but we mostly see it as part of the structure of the cell walls of bacteria. It’s no great mystery what type of organism an antibiotic against peptidoglycan would target then!

Viruses have neither a cell wall nor peptidoglycan. Let’s see if we can get a better answer choice.

Bacteria have a cell wall composed of peptidoglycan. Bonus points if you realized that an antibiotic targets bacterium!

Fungi can have cell walls, but they are not composed of peptidoglycan, so this can be eliminated. Answer choice B was a superior answer choice.
Protozoa are a subtype of Eukaryote, which do not have cell walls composed of peptidoglycan. That means our best answer here is going to be answer choice B: Bacteria.
Opiates, such as morphine and heroin, increase impulse traffic on the PS nerves to the iris. Therefore, after a morphine injection, the pupils will be more. At first glance, this question is related to the passage, but the passage was so content-heavy that there’s likely no need to go back to the passage for a lot of these questions. For question 44, I’ll focus on the effect of increased impulse traffic on the parasympathetic nerves to the iris. Light enters the retina, stimulates the optic nerve, and activates parasympathetic pathway to circular muscle of the iris. Constriction of the pupils happens when circular muscles contract. That means we’re expecting pupils will constrict.

First part of this answer choice matches my expected answer. But is the reasoning here correct? We have to think back to some details our general knowledge and from the passage: Acetylcholine and norepinephrine are the main neurotransmitters used in the parasympathetic and sympathetic divisions, respectively. Acetylcholine and parasympathetic go hand in hand. I’m liking this answer choice, let’s keep comparing.

Again, first part of our answer choice matches my expected reasoning. But what about our reasoning? Norepinephrine is the main transmitter used in the sympathetic nervous system, not parasympathetic like the question stem mentions. We can eliminate answer choice B for contradicting the passage. We’re keeping our superior answer for the time being, answer choice A.

This answer choice contradicts what the author says in the passage. Circular muscle of the iris is innervated by the parasympathetic division, and contraction will lead to constriction, not dilation.
Similar to answer choice C, we expect the pupils to constrict. We also expect the release of acetylcholine-the main neurotransmitter used in the parasympathetic nervous system. That means we can eliminate this answer choice and we’re left with our correct answer, answer choice A.

Drugs affecting the ANS may have either an “active” mechanism (that mimics the activity of the PS or the S division) or a “passive” mechanism (that blocks the effect of the opposing division). The drug atropine prevents acetylcholine from attaching to its receptors on the circular muscle. Is this mechanism active or passive? Put more simply: does atropine work by mimicking a division of the autonomic nervous system, or by blocking the effect of a division? The test maker gives us a definition of active and passive mechanisms, so we have to decide how to classify atropine. Atropine prevents acetylcholine, which I just mentioned is the main transmitter in the parasympathetic nervous system. It prevents acetylcholine from attaching to its receptors. What does that mean? We’re not mimicking activity, rather we’re blocking the effect of the parasympathetic division. Just going by the definition and our association of acetylcholine with the parasympathetic division. We can say atropine’s mechanism is passive.

The “passive” part matches our prediction. Let’s look at the reasoning. Acetylcholine is the main neurotransmitter used in the parasympathetic nervous system. The effect of acetylcholine is what’s being blocked by attaching to acetylcholine receptors. Reasoning here sounds consistent with the initial breakdown I did of the question.
This answer choice starts well, but then jumps into sympathetic innervation. We just mentioned Acetylcholine is the main neurotransmitter used in the parasympathetic nervous system, so our reasoning here contradicts my breakdown and what the author says in the passage. Answer choice A remains superior here.
Both parts of this answer choice contradict my breakdown of the question. The question stem mentions an active mechanism mimics the activity of an autonomic nervous system division. Do we have that with atropine? No-we’re blocking the effect of the parasympathetic division. We can eliminate answer choice C also and answer choice A remains superior.
This is similar to answer choice C. Atropine’s mechanism is passive, and we’re also dealing with the parasympathetic nervous system, not sympathetic. We can eliminate answer choice D as well. We’re left with our correct answer, answer choice A. Atropine’s mechanism is Passive, because PS innervation is blocked.

If acetylcholine is removed from the circulation faster than norepinephrine is, which of the following autonomic processes would be most rapidly inactivated? In other words, this question is asking the effect of having more norepinephrine in circulation than acetylcholine. Essentially meaning more sympathetic division activity than parasympathetic. And let’s be careful because we want to know which process would be inactivated. The best answer choice is going to be something we associate with the parasympathetic division. This is almost like a standalone question, but we still want to use critical thinking to break this down. Acetylcholine is the main transmitter used in the parasympathetic division, while norepinephrine is the main neurotransmitter used in the sympathetic division. Meaning if we have more norepinephrine and less acetylcholine, we’re expecting inactivation of parasympathetic stimulation. Acetylcholine effects include more rest and digest, and less energy expenditure. Some specifics include during periods of rest, it slows the heart rate, lowers blood pressure, stimulates digestion, and moves blood flow back to the skin.

This answer choice contradicts our prediction, and the passage. We’ve mentioned throughout the passage, signals from the sympathetic system to radial muscles cause the pupils to dilate. We’re looking for an answer we associate with the parasympathetic division.
This sounds more like sympathetic stimulation. Sympathetic nervous system controls the body’s automatic response to danger, including dilating blood vessels. We’re worried about skeletal muscles having proper blood flow in a fight or flight situation. Both A and B contradict our prediction.
We actually anticipate the heart beating faster and an increase in blood pressure. This also contradicts what I said in my breakdown of the question.
This answer choice matches our prediction. We said sympathetic nervous system controls the body’s automatic response to danger, and that includes slowing digestion. Autonomic we associate with “rest and digest.” This answer choice is consistent with our prediction. We can eliminate answer choices A-C, those all contradicted our prediction. We’re left with our best answer, answer choice D: Stimulation of digestive secretions.

The autonomic nerve fibers that directly innervate the heart to cause cardiac slowing are. This is another question where we don’t have to focus as much on details from the passage. This is almost like another standalone question where we’ll explain which autonomic nervous system division correlates to cardiac slowing. We’re deciding between parasympathetic and sympathetic correlating to cardiac slowing. The parasympathetic nervous system works in opposition to the sympathetic: during periods of rest, the parasympathetic nervous system slows the heart rate. That means we’re looking for an answer choice consistent with parasympathetic fibers.

We’ve run into a small issue right away. I did break down the question and an educated guess about which division of the autonomic nervous system we expected to see in our correct answer. However, I did not distinguish between motor and sensory fibers. There’s no need to get flustered or panic if that initial guess doesn’t match the format of the answer exactly, and instead let’s do a quick overview:
The CNS is the main control center of the body—it takes in sensory information, organizes and synthesizes this input, then provides instructions for motor output to the rest of the body. That means if we’re sending information to the heart, we’re dealing with motor fibers. This answer choice is partially consistent with the initial breakdown I did of the question, and this information about sensory vs motor fibers.

Just like answer choice A, we have sympathetic fibers, but now we’re given sensory fibers. We said sensory fibers take information into the CNS, not an output of instructions. This answer choice contradicts the definition provided in the question stem, so answer choice A is superior.

This answer combines the correct part of answer choice A (the motor fibers) with parasympathetic instead of the sympathetic division. That’s exactly what I said during the breakdown of the question. This is a superior answer to answer choice A which contradicts that breakdown. Answer choice C is now our best answer choice.

This answer choice mentions sensory fibers, not the motor fibers that are needed to cause cardiac slowing. Another contradiction here, so I’m also going to eliminate answer choice D. We’re left with our correct answer, answer choice C: parasympathetic motor fibers.

Based on information in the passage, would the S or the PS division of the ANS be expected to produce a more rapid systemic (whole-body) response to a stimulus? We’re looking for a specific response, and the response has to be quick, and throughout the body. We’re going to focus on the structure of the two divisions the author presented early in the passage. I want to pull up part of the passage here to make sure the breakdown of the question is clear:

I want to bring our attention to something near the middle of this paragraph. It says: the S ganglia appear near the spinal cord in the thoracic and lumbar regions and connect with each other to form the sympathetic trunk. Then soon after we have the PS ganglia lie in or near the organs they connect to but do not connect with each other. What does this tell us? When we’re trying to produce a rapid systemic response, we want as many pathways as possible. The sympathetic ganglia connect with each other, meaning exactly what we’re looking for. More pathways by which we can get a response to a stimulus. That’s the biggest difference the author presents to us in the passage.

At first glance, this answer choice is consistent with the breakdown of the question. We said more pathways to ganglia mean our response can be more rapid. Structurally, that’s the biggest difference the author gave us that would produce a more rapid response.
The main neurotransmitters the author presented in the passage did not represent a difference in speed in producing a systemic response to a stimulus. The author didn’t mention secretion of norepinephrine represented a more rapid response. Answer choice A remains superior.

This answer choice represents the opposite of the breakdown I did of the question, and what we want from our answer. The ganglia not being interconnected implies there are fewer pathways to ganglia, and we’ll have more localized responses.

This answer choice is similar to answer choice B. It’s not addressing the specific question being asked. The main neurotransmitters the author presented in the passage did not represent a difference in speed in producing a systemic response to a stimulus. That means we’re left with our correct answer, answer choice A: The S division, because its ganglia are interconnected.
The ocular drug physostigmine inhibits acetylcholinesterase, the enzyme responsible for the hydrolysis of acetylcholine. Administration of physostigmine would be expected to cause the pupil to. If we have less hydrolysis (or breakdown) of acetylcholine, what happens to the pupil? There’s more physostigmine which inhibits the enzyme acetylcholinesterase. That enzyme normally hydrolyzes acetylcholine. What does that mean? We’re breaking down acetylcholine. Therefore, if we inhibit the enzyme acetylcholinesterase, we’re not getting that same breakdown of acetylcholine. No breakdown means we have elevated acetylcholine levels.
At this point I’m combining some aspects of other questions and what the author says in the passage to break down what elevated acetylcholine levels mean. Acetylcholine is the main neurotransmitter used in the parasympathetic nervous system. When we’re in a more “rest and digest” situation, we have stimulation of the parasympathetic division, and constriction of the pupils when circular muscles contract.
Both parts of this answer choice contradict the breakdown I just did. We expect the pupil will constrict, and that’s because of increased acetylcholine levels and stimulation of the parasympathetic division.
First part of our answer choice is incorrect, and the reasoning contradicts the answer we’re looking for as well. We expect constriction, and constriction would happen due to increased acetylcholine levels. This is still a better answer choice than answer choice A, because it correctly identifies there’s increased acetylcholine levels.
First part of our answer choice is correct. We’re expecting constriction of the pupil, but the reasoning here is incorrect. We’re expecting constriction due to increased acetylcholine levels, not decreased.
This answer choice is consistent with what I said we’re looking for in the breakdown of the passage. We expect increased acetylcholine levels which ultimately means constriction of the pupils when circular muscles contract. We can eliminate answer choices B and C because both partially contradicted our prediction. We’re left with our correct answer, answer choice D. The pupils would constrict, due to increased acetylcholine levels.

The confining membrane shown in Figure 1 is most similar structurally to a. To answer this question, we can flip back to the passage and look at Figure 1. We’ll note the membrane structure.

We want to note what the author says about the lipid bilayer. The passage says “Researchers’ further analyses of the objects showed them to consist of RNA and core protein wrapped in a phospholipid bilayer membrane from which proteinaceous knobs protruded.” What might this remind you of right away? A plasma membrane with a phospholipid bilayer. Not just the phospholipid molecules, but also the proteins. Think of integral proteins and how they’re integrated into the membrane structure. We have a similar structure here. That means we’ll try and compare this confining membrane to our traditional plasma membrane structure we just described.

This answer choice matches what I said in the breakdown. Eukaryotic cell membranes have that same phospholipid bilayer and integral proteins, like we mentioned. We have a very similar structure, so we’re liking answer choice A for the time being.
The prokaryotic cell wall is made of thick, rigid peptidoglycan. It’s meant to protect the cell’s interior and resist mechanical pressures. That structure is much more different than the structure in Figure 1, at least compared to answer choice A. We’ll stick with answer choice A as the superior answer choice for now.

The spore coat is made of keratin and proteins. There are multiple layers, and beneath the coat we have the cortex and germ cell wall. That’s not the structure we see in the confining membrane in Figure 1. We’re still sticking with the more obviously similar answer choice-answer choice A.

Bacterial capsule is a polysaccharide structure outside of cell membranes. These capsules are meant to protect bacterial cells and will even cover the cell wall. Structure in Figure 1 is a phospholipid bilayer, which more closely matches answer choice A. That means we can eliminate answer choice D. We’ll stick with our correct answer, answer choice A: eukaryotic cell membrane.

Which of the following conclusions regarding virion biology is supported by information given in the passage? We’ll have to make connections to what we know about virions in general and what the author says in the passage. I can pull up an excerpt one more time.

We’re not going to spend a ton of time revisiting the passage because it was so short. Let’s break down any observations we can make here. Virions are 100 nanometers in diameter. They have RNA and core proteins, confined by a phospholipid bilayer membrane. There are no organelles, and attempts to grow virions in a noncellular growth media were unsuccessful. That means the virus can’t reproduce without a host cell.

This answer choice contradicts what the author says in the passage. The virions consist of RNA and core protein. We also see that in our Figure 1. The author makes no mention of virions lacking genetic material either.

Another answer choice the author doesn’t address directly. One clue we do have though is the virions don’t have organelles. Without ribosomes of their own, virions can’t synthesize proteins. This also contradicts my breakdown of the question.
This answer choice is also contradicted in Figure 1. We saw the enzyme reverse transcriptase in the confining membrane, so that means so far choices A-C have all contradicted the passage.
An obligate parasite is a parasite that cannot reproduce or grow without a host. This sounds exactly like what I said in my breakdown. I said attempts to grow virions in a noncellular growth media were unsuccessful. These virions need host cells. This is going to be our best answer choice. Answer choices A, B, and C all contradicted information from our passage. We’re left with our best answer, answer choice D: Virions are obligate parasites.

In regard to their relative size, the described objects are. The author is a little ambiguous with the verbiage here. We want to explain the relative size of the spherical objects in the passage: the virions.

I’ve pulled up part of our passage here again, and we want to focus on what the author mentions about size. First thing we want to note, we couldn’t detect the spherical shaped objects with a light microscope. We had to use an electron microscope. We’re also told the objects, the virions, were estimated to be 100 nanometers in diameter. That’s a very small number, but we might not know what that means relative to other cells. What we do know is we can see eukaryotic cells in light microscopes. That means these virions are much smaller than eukaryotic cells, and we could only see them with the higher resolution microscope.

This answer choice sounds good right off the bat. We know we can see eukaryotic cells using a light microscope, but we need an electron microscope to see the described objects. What does that tell us? Virions are smaller than known eukaryotic cells. Good start here with answer choice A.

This is an answer choice we’ll have to explain using our general knowledge. We’re going to compare the size of bacteria versus the size of a virus. In general viruses are much smaller than bacteria. Think about it this way, a virus isn’t even a full cell-it needs a host just to survive. Viruses are tiny-they can be tens to hundreds of times smaller than the smallest bacteria, so we can eliminate answer choice B.

Red blood cells can be seen with a light microscope and are also going to be much larger than viruses. This is similar to answer choice B, so answer choice A remains the superior answer so far.

We can reason this answer out one step at a time. Bacteriophages are viruses in bacteria. If a virus can fit inside the bacteria, then the viruses by themselves are naturally going to be smaller than bacteria. That means we’re left with our correct answer, answer choice A: smaller than all known eukaryotic cells.
Which of the following media would most likely be used to grow virions in the laboratory? Viruses don’t have any specialized machinery to produce energy, grow, reproduce, or maintain homeostasis by themselves. That’s why they take over host cells-to use the host and specialized organelles to replicate, and make new viral particles. That’s why the attempts to grow the objects in a noncellular growth media were unsuccessful. The virus can’t reproduce without a host cell, so we’re looking for an answer choice that takes this information into account.

This answer choice is not consistent with our prediction. We need actual host cells so the virus can survive and reproduce. Not possible with just a suspension of ribosomes and ATP.

This answer is similar to answer choice A. Again, we need full host cells so the virus can survive and reproduce. We can’t see growth in virions unless the virus can replicate and live off a host cell. We’ll still keep answer choices A and B.

Even if there was a nutrient broth, the virus can’t utilize it to survive and replicate. We need an answer choice that matches our prediction exactly, because that’s the only way we expect the virions to survive and grow.

This answer choice is what we were looking for. When we have a fragment of tissue, or even a small population of cells, the virus can use these cells as hosts. The viruses can use the host and specialize organelles to replicate and make new viral particles. The virus will be able to reproduce. We can eliminate answer choices A-C. None of those were viable answers, and viruses wouldn’t grow in those media. We’re left with our correct answer, answer choice D: A tissue culture.
Most viral proteins are produced directly by.
Central dogma explains how information in genes flows into proteins. DNA to RNA by transcription. RNA to protein by translation. We can elaborate a bit here. Transcription is the process of converting a specific sequence of DNA into RNA. mRNA is formed from one strand of DNA. Translation is the process where a ribosome decodes mRNA into a protein. We can reference a picture that allows us to visualize the central dogma and see how everything is connected.
First part of this answer choice is consistent with my breakdown and the central dogma. Translation produces proteins. But we’re asked about viral proteins, not host proteins. That’s the only thing wrong with answer choice A.
Similar to answer choice A here. We have translation, which is the process where we decode RNA into a protein. And this answer choice is talking about viral proteins, not host proteins, which is exactly what we want. This is a better answer choice than answer choice A because answer choice A incorrectly mentioned the host nucleic acid.
This answer choice incorrectly mentions transcription instead of translation. Transcription is the process of converting a specific sequence of DNA into RNA. That’s not what we’re looking for. We’re asked about production of viral proteins. Answer choice B remains superior.
This answer choice also incorrectly mentions transcription instead of translation. We said transcription is the process of converting a specific sequence of DNA into RNA. Translation is what we were looking for-we wanted RNA being translated into protein. We can stick with our only answer that’s factually correct and consistent with what we need to answer this question. Answer choice B: Translation of viral nucleic acid.
A microbe pathogen was hypothesized as the causative agent of the disease described in the passage because:
I. suspicious objects were found in blood samples from ill patients.
II. in vitro cultivation of the probable pathogen was difficult.
III. the disease was infectious.
One thing to always keep in mind with questions like these is there can be more than one correct answer. That means we’re going to decide which of these three could be the reason the pathogen was suspected as the cause of the illness. This answer is going to be a hybrid. We’ll have to know how viruses operate-meaning how a patient would present if infected, or how a virus can grow. That will come from our general knowledge. We’ll also get clues from the passage as needed. For now, we can break down each of the three options and see if it’s a viable option to explain why a microbe pathogen was hypothesized as the causative agent of the disease.
Choice I says suspicious objects were found in blood samples from ill patients. Think back to the passage. The author mentioned the microbial agent was suspected as the cause of the diseases, but the agent couldn’t be found under a light microscope. Despite not seeing the microbial agent, the agent was still a suspected cause. Option I is going to be incorrect, and what does that mean in terms of this question? We can eliminate answer choices C and D right away. And only one of option II and III are correct.
Option II says in vitro cultivation of the probable pathogen was difficult. What does in vitro mean? Outside of the living body. We know viruses can’t survive without a host cell in general, so option II is not possible. At this point we can go ahead and pick answer choice B because it lists option III, but we always want to be thorough. We’ll go through option 3 and confirm it’s consistent with what’s asked in the question stem.
Option III says the disease was infectious. This is consistent with what the author mentions in the passage. This was the first guess given by the author, and it’s the hypothesis we’re looking for as our best answer choice. Ultimately, we know the cause of the disease was actually a virus, but option III was the reason for the initial hypothesis. Like I mentioned, there’s only one option here that matches option III only, and that’s answer choice B. Whenever possible, go through these Roman numeral questions exactly like I did here. For example, just by eliminating option I, we were able to eliminate two of the four answer choices. Take any opportunity you can to get the numbers in your favor.

Which of the following statements about pulmonary function best describes all of the results graphed in Figure 1? This question is going to come from our passage, so I can pull up Figure 1 from the passage:

We have Figure 1 from the passage here. This is a graph that’s allowing us to compare the global score of pulmonary function for three different patients. We see a sharp decrease in pulmonary function initially. All of the patients hit a global score of as low as 50. After hitting that low, each patient here seemingly recovers slowly. It takes more time to get back to that normal global score, but each patient eventually gets back. What are our variables? X-axis is days of illness. And as the days go by, we measure the global score of pulmonary function of each patient. Biggest trend is the sharp decrease in global score in the first few days of illness, then a slow climb back up. All of the 3 patients recover and get back to a normal level.

One-half normal is a global score between 200 and 225. Each patient is below that threshold for fewer than 7 days total. This answer choice contradicts Figure 1 above. Not a great option to start.

This answer choice is only true for one patient. Patients 1 and 2 saw a decline earlier than day 4. This answer choice is including all 3 patients, but is only true for patient 3. Neither A nor B are great options so far. Let’s keep looking for something better.
This answer choice matches what I said in my breakdown of the question. The big trend we saw was a sharp decrease in global score in the first few days of illness, but there was a much slower recovery that took more days than the initial decline. We can now eliminate answer choices A and B. I said both contradicted Figure 1 from the passage, and now we have a superior answer in answer choice C.
This answer choice mixes up the speed of the decline and recovery. We see a much quicker decline initially that’s followed by a slower recovery. We can eliminate answer choice D because it also contradicts the figure from the passage. We’re left with our correct answer, answer choice C: It showed a rapid decline, followed by slower recovery.

What was contained in the sera from the respiratory patients of Experiment 1 that caused the sera to react with a hantavirus that causes kidney disease? This answer is going to come from Experiment 1 in the passage. Why did the seta react with that kidney-disease causing hantavirus? Let’s recall what we saw in the passage:

I brought up the part of the passage that goes over Experiment 1 here. It says A positive immunologic reaction was seen with a hantavirus that causes kidney disease. First of all, what does this mean? The pathogen has antigens that are the same, or similar to, something already known: a hantavirus that causes kidney disease. We know we have a different ailment: we’re not dealing with kidney problems, we’re dealing with respiratory problems. So, we may not have the exact same mechanism or effects, but again: we have similar antigens-we’re getting the same immune response.

Let’s break down what this answer choice means. When the patients were exposed to the pathogen, antibodies are produced-we now have antibodies for the unknown pathogen in the bloodstream. When the serum is exposed to a known virus, there’s a reaction: the serum contained antibodies for antigens found on that known virus. That helps the experimenter determine the unknown virus has similar antigens. The sera contain antigens for the unknown pathogen, not the kidney disease hantavirus itself. Another thing I also mentioned when breaking down this question, we’re not dealing with kidney problems or the specific kidney disease causing hantavirus. Rather we’re dealing with a different ailment-the unknown pathogen that causes pulmonary disease. Those are the antibodies we’re expecting in the sera. Ultimately, the unknown pathogen is related to hantaviruses, or a different strain than the kidney disease-causing hantavirus.
This answer choice is similar to my breakdown of the question. And we’ll also use the breakdown I just used to analyze option A. We mixed the patients’ sera with known pathogenic viruses and bacteria. Those sera have antibodies to the antigens of the unknown virus. That’s how the immune system works. We had an immunological response with a hantavirus that causes kidney disease because of similar or identical antigens to the pathogen. Answer choice B is superior here.
This answer choice contradicts what I said in my breakdown and what we know about immune responses. The sera have antibodies to the unknown pathogen. There’s no mention in the passage, and there’s no way to conclude there are antibodies to kidney proteins in the sera. Answer choice B is still the superior answer.

The unknown pathogen is not the one reacting with the hantavirus. It’s the antibodies that are reacting. This answer choice is factually incorrect and it contradicts the passage. We can also eliminate this answer, so we’re left with our correct answer, answer choice B: Antibodies to the unknown pathogen, which is antigenically related to the known hantavirus.

In Experiment 2, why did the synthetic hantavirus gene sequences hybridize with nucleic acid from an infected lung? Let’s quickly revisit Experiment 2 from the passage and see how we should approach this question.
We want to know why hybridization occurs. We have Experiment 2 above, it says: Hybridization between nucleic acid strands occurs when they base-pair with each other. That means if we see hybridization between the nucleic acid strand from the pathogen and the gene sequences from the known hantaviruses, we have complementary sequences-we have base-pairing. And what would that tell us? It would give us even more evidence that the pathogen is closely related to the two known hantaviruses in Experiment 2.
This answer choice incorrectly says the sequences were identical. If the sequences were identical, we wouldn’t have base-pairing. We have hybridization when we have complementary sequences that are able to base-pair.

This answer choice matches the breakdown I just provided, and it’s congruent with the reasoning from when I analyzed option A. We have hybridization when we have complementary sequences that are able to base-pair. We can eliminate answer choice A that mentioned identical sequences. That was factually incorrect.

This answer choice isn’t really addressing the hybridization between the gene sequences and nucleic acid. It’s focusing incorrectly on an RNA-encoded protein from the infected lung. There was no mention of these proteins hybridizing with the hantavirus gene sequences. We can eliminate answer choice C because it’s making an assumption and a conclusion we can’t logically make.
Even if this answer choice were factually incorrect, it’s not consistent with what how the author defines and explains hybridization. The author says hybridization occurs between nucleic acids when they base-pair with each other. That means answer choice D is inconsistent with the passage. We’re left with our correct answer, answer choice B: They were complementary in sequence to a gene from the lung.

Why did investigators conclude that the new pathogen is a hantavirus? This answer is going to come from Experiments 1 and 2. We’re going to focus on the results of the two experiments and how they support the conclusion that the new pathogen is a hantavirus.

We have the first two experiments here, and we’re going to quickly break down the results. Experiment 1: we have a positive immunologic response with a hantavirus. That shows we have similar antigens-we’re getting the same immune response.
Experiment 2: we have hybridization between the synthesized gene sequences from two known hantaviruses and nucleic acids from the patients’ lung tissues. That gives us even more evidence that the pathogen is closely related to the two known hantaviruses in Experiment 2. Using these results, investigators came to their conclusion that the new pathogen is a hantavirus.

This answer choice is almost verbatim what I just mentioned in the breakdown. The results of Experiment 1 and 2 showed the pathogen is related to known hantaviruses so we were able to conclude the new pathogen is a hantavirus as well.

This was the result of Experiment 3. This is technically a true statement, but this doesn’t exactly answer the question being asked here. There are many pathogens that can affect lung endothelium. Just knowing the new pathogen affects lung endothelium isn’t enough to conclude the pathogen is a hantavirus. Answer choice A is still superior.
This answer choice contradicts the passage. This new pathogen affected the lungs, while the known hantavirus we were introduced to in the passage affected the kidneys, not the lungs. We can eliminate this answer choice and answer choice A still remains our best answer.
This answer choice makes an assumption that isn’t necessarily true. Just because we don’t have pathogenic bacteria in the infected tissue, does not mean we can automatically assume we’re dealing with a hantavirus. We might be able to conclude we’re dealing with a virus, but there’s no way we can call it a specific type of virus just by eliminating the potential of pathogenic bacteria. We’re left with our correct answer, answer choice A: The experiments showed that it is related to known hantaviruses.

The investigators concluded that the new hantavirus infects lung endothelial cells. Do the data support this conclusion? This going to be similar to the previous questions. We’re going to revisit Experiment 3 and determine if we have the proper data to conclude the new hantavirus infects lung endothelial cells.

Above I’ve got Experiment 3, and we’re focusing on the results. It says: These antibodies bound to capillary walls in patients’ lung tissues. What does that tell us? When the patients were exposed to the pathogen, antibodies were produced. In this experiment we have pathogen-specific antibodies binding to lung tissues. That lung tissue is where we have our viral antigens.
That means we’re looking for an answer choice that’s consistent with the following: we have viral antigens in capillary walls of affected lung tissues. That’s why antibodies bound to these capillary walls.

This answer choice is not something that was covered in the passage. Viral replication wasn’t demonstrated in the three experiments. The only thing tangentially related is using viral proteins to make pathogen-specific antibodies in Experiment 3.

This answer choice is making some assumptions that we can’t determine are correct. We don’t know if hantavirus were hybridizing with DNA. We also can’t determine if lung endothelial cells are affected. Endothelial cells are usually the cell layer that lines capillaries, and that’s what we need to answer this specific question.

This answer choice is consistent with our prediction. We said we have viral antigens in capillary walls-or endothelium-of affected lung tissues. That’s why we had antibodies bound to capillary endothelium. This ends up being the best answer choice so far. I’m going to go ahead and eliminate answer choices A and B. Neither A nor B were directly discussed in the passage, and answer choice B made an assumption that was not apparent in the passage.
First thing we want to note, we did have data in Experiment 3 that support the conclusion in our question stem. We don’t like the first part of this answer. Also, this is similar to answer choice B. We went through all 3 of our experiments in the passage, and we didn’t determine viral antigens were associated with lung alveoli. We found viral antigens were associated with capillary walls, or endothelium instead. That helped conclude the new hantavirus infects lung endothelial cells. We’re left with our correct answer, answer choice C: Yes; viral antigens were found associated with capillary endothelium.
Assuming that the breathing rate is 10 breaths/min, the tidal volume is 800 mL/breath, and the nonalveolar respiratory system volume (dead space) is 150 mL, what is the net volume of fresh air that enters the alveoli each minute?
This is not a traditional biology question, rather it’s more of a physics question in disguise. Luckily for us, we have to know both for the exam! As with all physics calculations, we want to tackle this problem by multiplying out the units to see if we can reach an appropriate answer choice, if possible.Noticing that each breath brings in 800 mL of air, but that 150 mL of that won’t enter the alveoli, we can say that 800 mL – 150 mL = 650 mL of fresh air are brought into the alveoli with each breath. However, we’re not done there. Note we’re not picking answer choice A at this point and moving on to our next question. We have to make sure we answer the specific question being asked.Next, notice the question stem mentions there are 10 breaths/minute, so we can multiply 10 breaths/minute by 650 mL air/breath:
10 breaths/minute* 650 mL air/breath = 6500 mL air/minute
This matches an answer choice exactly, and we can discount the other answers. Answer choice C is our best option.
In human females, mitotic divisions of oogonia that lead to formation of presumptive egg cells (primary oocytes) occur between: This question is all about timing, and we have to think in terms of human females. What we do we know about primary oocytes? Oogonia, the immature female reproductive cell, give rise to primary oocytes by mitosis. While a spermatogonium renews its population by mitosis throughout life, oogonium stops renewing its population sometime before birth. Make sure to not confuse mitosis and meiosis! If you think of this question in terms of meiosis by accident, you can easily get an incorrect answer.

This answer choice is consistent with what I said in our breakdown of the question. I mentioned oogonia give rise to primary oocytes by mitosis. This ceases some time prior to birth, so female babies are born with all of the oocytes they will need in their lifetime. This sounds like a good answer right from the get go, but we still want to compare with our other options.

We do expect mitotic divisions of oogonia to occur between fertilization and birth, but not all the way through puberty. Like I mentioned in the breakdown of answer choice A, female babies are born with all of the oocytes they will need in their lifetime. Answer choice A remains the best option.
This answer is the opposite of our breakdown. If the question were asking instead about when meiosis occurs, this would be a viable option. However, we’re concerned with mitotic divisions of oogonia that lead to formation of primary oocytes.
This is similar to answer choice C in that this is much too late in the timeline. We mentioned we expect an oogonium to give rise to primary oocytes by mitosis prior to birth, but not subsequently. Answer choices B-D were all incorrect, so we’re sticking with the best option here: Answer choice A.

The discovery that the amount of thymine equals that of adenine and the amount of guanine equals that of cytosine in a given cell provides supporting evidence that: We’re jumping into some biology/history fusion for this question! We’re given a discovery, and we’re asked to relate this discovery to something we likely use in modern biology. Which discovery are we focused on in this situation? DNA base pairing in which thymine pairs with adenine and guanine pairs with cytosine. What I want to be aware of here is something AAMC likes to throw into their test material. Oftentimes they will give us multiple correct statements in the answer choices to throw us off. What do I mean by that? We’re only looking for the answer for which the question stem is providing supporting evidence. Even if a statement is correct in a vacuum, it might not be supported by the discovery that the amount of T=A and A=G.

It’s fairly commonly accepted that DNA usually exists in a form described by the Watson-Crick model. In this structure, DNA is made up of two strands that are twisted around each other to form a right-handed helix, called a double helix. The strands lie side by side in antiparallel 3′ → 5′ directions and are bound together by the hydrogen bonds between nitrogenous bases, forming a double-stranded structure. This base-pairing takes place between a purine and pyrimidine: adenosine (A) with thymine (T), and guanine (G) with cytosine (C). This last part is where we want to focus. A pairs with T and G with C. That’s consistent with what we’re looking for in our question stem, so we like this answer choice. Let’s keep comparing.

Let’s break down what we know about DNA. DNA is a self-replicating material which is present in nearly all living organisms as the main constituent of chromosomes. It is the carrier of genetic information. Despite this being a true statement, does it answer this specific question being asked? That’s what I wanted us to be careful about. The amount of thymine equaling the amount of adenine doesn’t provide supporting evidence that DNA is the genetic material. Answer choice A is still our best option.

This is similar to answer choice B. This is a true statement and the genetic code is universal because it’s the same among all organisms. Once again, this doesn’t answer our specific question being asked, so we can eliminate this answer choice. Answer choice A remains superior.

Three nucleotides (triplet) in the genetic code encodes a specific amino acid (or stop signal). Despite the fact three nucleotides correspond to a codon, this doesn’t answer the specific question being asked either. Answer choice A is the only option that meets our criteria. We’re left with our correct answer, answer choice A.

Which of the following tissues have cells that are in direct contact with the external environment or elements of the external environment?
The lining of the reproductive tract
The lining of the respiratory tract
The lining of the gastrointestinal tract
To answer this question, we’ll have to consider all three options listed above. One thing I want to point out for this question: all three of the choices are listed in exactly three answers. If we find that any of the three options does not have cells in direct contact with the external environment or elements of the external environment, we’ll eliminate those three options and have our correct answer. Alternatively, if all three tissues have these cells, then D becomes our correct answer.
Right away you might notice a common denominator between all three options. Mucous membranes line the reproductive, respiratory, and digestice tracts. Why is that? It acts as a barrier and stops pathogens from entering the body. Think of nostrils and mouth, genital area, anus, and lips. Those are all places where we have some contact with the external environment or elements of the external environment. What does that mean for us here? All three of the answer choices match the criteria set forth in the question stem. All three have cells that are in direct contact with the external environment or elements of the external environment. That means we can eliminate answer choices A-C which only list two of the three options. We’re sticking with the best answer here: Answer choice D.
A hiker becomes lost and has no drinking water for 2 days. At the end of this time, which of the following changes in hormone production would be expected to be significant in this individual? A terrible situation for this poor hiker! I can’t imagine the suffering and fear that would take place in this situation, but also the physiological toll it would take. Two days without water is a significant amount of time (especially for a hiker or someone doing strenuous activity), so we also suspect there is some level of dehydration.
This question is very open-ended beyond that, but we want an answer that is consistent with the situation presented in the question stem and the short breakdown I just did of the question.

Glucocorticoids are a set of steroid hormones involved in glucose metabolism. The most important one for the MCAT is cortisol. In situations like the one presented in the question stem, glucocorticoids are helpful in restoring balance and mediating the stress response. We don’t expect decreased glucocorticoid secretion.

Aldosterone causes the tubules of the kidneys to increase the reabsorption of sodium and water into the blood. However, in this situation the hiker has no drinking water for two days and we said is likely dehydrated. The hiker needs to reabsorb as much water as possible in this situation, so we’re not expecting decreased aldosterone secretion.
Insulin is a polypeptide hormone produced by the pancreas. It regulates carbohydrate metabolism by lowering the sugar level in the blood. This is typically in response to eating food. While there’s no way to know about the food supply, this is still the best option so far. Answer choices A and B both suggested options that are the opposite of what we’d expect.
This answer choice sounds like a keeper. The intake of fluids is stimulated by specialized osmoreceptors in the brain that detect dehydration. These receptors communicate with the pituitary gland to stimulate the release of antidiuretic hormone, which in turn communicates with the kidneys to reduce urine production by reclaiming water. That’s exactly what you’d need in a situation where dehydration is an issue and there is a lack of water. That means we can pick our best answer choice: Answer choice D.

The most likely explanation for the difference in skin blood flow between the fatty acid group and the fatty acid + vitamin E group in Figure 1 is that. This answer requires details that come from our passage, so we’ll go back to Figure 1. We’ll see what the author says specifically about the effects of vitamin E.

Up top we have the part of our passage where the author mentions vitamin E. We also have Figure 1 showing the % change in blood flow in the 4 treatment groups. The passage says Vitamin E, an effective anti-oxidant, was given to 2 groups to reduce in vivo oxidation of the ingested fatty acids. Let’s break this down a bit further. Vitamin E is an anti-oxidant, meaning it will reduce in vivo oxidation of the ingested fatty acids. Fatty acid oxidation is the process of breaking down fatty acid molecules. Vitamin E is reducing that process, so we can tie that into our quantitative results. Let’s take a look at our graph. When fatty acid is supplemented without vitamin E, we have the largest % decrease in blood flow. The fatty acid plus vitamin E group actually saw a positive % change in blood flow. What does that tell us? Adding vitamin E and reducing fatty acid oxidation will increase skin blood flow. An absence of vitamin E and increased oxidation decreases skin blood flow.

This answer choice is factually incorrect and contradicts the figure in our passage. Vitamin E alone increases skin blood flow significantly compared to fatty acids alone.

Unlike answer choice A, this answer choice is factually correct, but does it answer the specific question being asked? We want to know the explanation for the difference. Looking at the results of the vitamin E alone can be informative, but not to answer this specific question. The explanation is most likely going to have to do with the reduction in fatty acid oxidation when we have both vitamin E and fatty acids supplemented together. We’ll still keep answer choice B for comparisons. That means we can eliminate answer choice A because it was an inferior option.
This answer choice is factually correct, and it provides a viable explanation to our question. We want to explain the difference in skin blood flow between the fatty acid group and the fatty acid plus vitamin E group. The difference in supplements is the vitamin E, which the author says is an effective anti-oxidant. That anti-oxidant effect if what causes skin blood flow increase. If we don’t have that effect and instead we have normal fatty acid oxidation, we have a reduction in skin blood flow. We can keep this answer choice over answer choice B. Even though it was factually correct, answer choice B didn’t give us a proper explanation.
This answer choice is factually incorrect. Vitamin E is an anti-oxidant and when it’s present, we have increased blood flow. What does anti-oxidant presence mean? We have unoxidized fatty acids. We wouldn’t expect reduced skin blood flow. This answer choice contradicts Figure 1. That means we’re left with our correct answer, answer choice C: the products of fatty acid oxidation reduce skin blood flow.
It was hypothesized that the decrease in blood flow to the skin resulted from a change in the activity of the sympathetic nerves to the skin. Which of the following observations would support this hypothesis? Let’s break down how I’m going to approach this question. First, I am going to rely on my knowledge of sympathetic nerves and blood flow. Specifically, I want to consider how blood flow is regulated by the autonomic nervous system.
A decrease in blood flow will mean constriction of skin blood vessels. How might we be able to tell this is a change in activity of sympathetic nerves? The biggest thing is norepinephrine is the main transmitter used in the sympathetic nervous system. Acetylcholine is the main neurotransmitter used in the parasympathetic nervous system. This might be the only way to distinguish between nerves and specifically support the hypothesis.

This answer choice actually matches our prediction. We said norepinephrine is the main transmitter used in the sympathetic nervous system. If we have a change in norepinephrine content and concurrent change in blood flow, that would support the hypothesis in our question stem.

Acetylcholine is the main neurotransmitter used in the parasympathetic nervous system. This answer choice would directly contradict the hypothesis given in the question stem. We can eliminate answer choice B.
A change in receptor numbers doesn’t indicate there’s a change in activity of sympathetic nerves to the skin. This could correlate to the decreased blood flow, but wouldn’t support the specific hypothesis in the question stem. Let’s eliminate answer choice C.
Dilation of blood vessels means widening and increased blood flow. Also, dilation of skin blood vessels is related to the activity of parasympathetic nerves. This answer choice contradicts the question stem as well. We can eliminate answer choice D. We’re left with our correct answer, answer choice A: change in the norepinephrine content of blood draining from the skin.
An alternative method for examining the effects of fatty acids on blood flow would be to measure changes in blood pressure. If blood pressure were measured, one would predict that it would be lowest in which of the following? This is almost like a standalone question that’s tangentially related to the topics covered in the passage. We’re going to note the differences in blood pressure at different parts of the circulatory system. We can list out the order in which blood pressure goes from highest to lowest in the body:
Blood pressure is highest when it leaves the heart and goes through large arteries. Next is going to be small arteries and arterioles. Mid-level pressure is going to be capillaries. And the lowest would be venules, veins, and vena cava.
This is going to be the highest blood pressure according to my breakdown of blood pressure at different points in the body.
Arteries have high blood pressure, but this is going to be a better answer than answer choice A. Let’s keep our superior answer choice for the time being: answer choice B.
Again, as simple as looking at our breakdown and noting arterioles have lower blood pressure than arteries. We can eliminate answer choice B; answer choice C is now the best answer choice.

Capillaries will have lower blood pressure than arterioles (which was our previous best answer). One thing I do want to point out is this was our best answer, but we could easily have listed veins and had that be the correct answer instead. We’re always picking between the 4 options given, even if they don’t exactly correspond to the absolute best option in our breakdown or if there’s a better, unlisted option out there.

To interpret the results, the researchers must assume that. At first glance, this question’s a bit different than what we’re used to from AAMC. It’s so open-ended that we have to go through our answer choices to get a sense of what the author is asking. The researchers could be assuming an almost infinite number of things, so it’s nearly impossible to come up with something that’s exactly addressing what the author wants us to pick. You’ve noticed that normally I don’t like doing that. The reasoning behind that is because we end up getting biased and leaning toward an answer choice before we get a chance to break down and answer the question on our own. In this case, it’s the only way.
This ties into the purpose of our experiment. The entire purpose of the research is to see the effect of fatty acids on skin blood flow. This answer choice is unreasonable. There’s no way the researchers would assume fatty acids have no effect if they’re trying to find the actual effects of fatty acids on skin blood flow.

The researchers actually tested this in the experiment-they didn’t assume this. Vitamin E was actually given to 2 of the treatment groups. In the end, vitamin E actually increased skin blood flow.

This answer choice ties into what the researchers asked of the subjects. The subjects were instructed not to alter their habits or lifestyles during the experiment. The researchers have to assume the subjects listed and didn’t alter their habits. Let’s hold on to this answer choice and eliminate answer choices A and B. Neither A nor B reflect what happened in the experiment.
The only difference in blood pressure would be a direct result of a change in blood flow. The researchers tried to take out any other possibilities that could alter or influence the experimental data. We can’t assume blood pressure differed, because that ties into what we’re testing and we’re keeping all other variables as close to the same as possible. That means we’re left with our correct answer, answer choice C: the subjects did not alter their habits during the study.
In the design of the experiment, all of the following factors were controlled EXCEPT. This is actually going to be similar to our last question. The question is very open-ended, so we’re going to do a quick overview of the controlled factors and then jump right in. We’re focused on the methodology and we want to know which of the factors listed were NOT controlled. Let’s break down the experiment. Subjects were all males within a 10-year age range and they were all screened beforehand and habits weren’t altered. Dosages of fatty acids and vitamin E were regulated, and no subject knew which supplement they were ingesting. The experiment was 60 days for every subject and blood flow was measured at a set temperature, set time of day, and in the same environment. Subjects all fasted for the same amount of time.
This answer choice contradicts our passage. Skin temperature was 32 degrees for all of the subjects. This was controlled, so not a great option so far.

Age of the subjects was between 18 and 28. The subjects were divided randomly. This is also a controlled factor, so we’re 0/2 here. Neither A nor B is a great option for this question.

The subjects all got tested at the same time, and in the same environment. They were all required to fast for at least 12 hours and rest prior to measuring. We’re not expecting any variation in diurnal rhythm because these factors were all controlled and kept consistent.
This is what the researchers were looking for throughout the experiment. Controlling skin blood flow would defeat the point of the experiment. The variation in skin blood flow is what allowed the researchers to make a conclusion about the effects of fatty acids on blood flow and the effect of anti-oxidants on blood flow. Answer choice D is the factor being tested for and tracked. It’s not something controlled during the experiment. The author explicitly mentions controlling answer choices A-C, so answer choice D is our best option.
Assuming Hypothesis B to be correct, which of the following endocrine disorders would cause hypertension that could NOT be rectified by physiologically normal kidneys? There’s a lot to unpack here, so the way I want to start this question is by restating the question in simpler terms that make sense to me. We want to know which of the endocrine disorders listed in the answer choices wouldn’t be able to be fixed by the kidneys, and lead to hypertension. Something else we want to note: this is assuming hypothesis B is correct, meaning kidneys can correct increased blood pressure.
We’re just going to be looking for an answer that would cause hypertension, even if kidneys are functioning properly. We can break down the pathway: renin is secreted by kidneys and results in the formation of angiotensin II and increased blood pressure. Angiotensin II stimulates secretion of aldosterone, which causes the tubules of the kidneys to increase reabsorption of sodium and water into the blood. That increase of volume also increases blood pressure. Even if kidneys were functioning properly, that would still mean increased blood pressure. The kidneys don’t control anything antagonistic to aldosterone, so excess aldosterone and hypertension won’t be rectified.
I touched on aldosterone in my breakdown. Aldosterone causes the tubules of the kidneys to increase the reabsorption of sodium and water into the blood. This increases the volume of fluid in the body, which also increases blood pressure. If we have an excess of aldosterone, we’d have increased blood pressure and hypertension, especially if the kidneys are working properly. I am liking this answer choice for now
What’s the role of glucagon? Glucagon stimulates the breakdown of glycogen when blood glucose levels are low. Excess glucagon would mean excess glucose in the bloodstream, and not enough being stored as glycogen. Excess glucose is not going to directly cause increased blood pressure. The kidneys major roles are fluid balance and blood pressure-these functions would still be working. We’re still sticking with the most direct answer which is answer choice A.
Thyroxine is important in the body’s metabolism, digestion, and heart function. The answer choice isn’t directly related to kidney’s function, and won’t prevent the kidneys from regulating body fluid. Answer choice A is still our superior answer, because excess aldosterone will actually increase blood pressure with a properly working kidney.
This is going to be the same effect as answer choice B. We’d have excess glucose in the bloodstream. But kidneys would still be functioning to maintain fluid balance and blood pressure. Just like we did with answer choice B, we can eliminate this answer choice and stick with the superior, more direct answer: answer choice A-An excess of aldosterone
What mechanism probably would be responsible for the increased urine output induced by hypertension according to Hypothesis B? First thing we want to do is note we’re focused on Hypothesis B. Remember, hypothesis B says kidneys can correct increased blood pressure. We’re going to explain how the kidneys function to restore proper blood pressure by increasing urine output. The formation of urine takes place in the nephron, and as we talk through it, we can visualize as well with this figure:Formation of urine happens through three steps: glomerular filtration, tubular reabsorption, and tubular secretion. Let’s do a quick overview and we’ll jump into our answer choices. Glomerular filtration filters out most of the solutes. And glomerular filtration rate (GFR) is the volume of glomerular filtrate formed per minute by the kidneys. Increased urine output induced by hypertension means increased GFR.
Second step is tubular reabsorption in the proximal convoluted tubule. We have reabsorption like the name suggests-that means nutrients, electrolytes, and water if there’s lower blood pressure. For this question, we’re looking for an answer that either doesn’t increase absorption of water, or increases urinary output.
Increased blood flow to the bladder does not necessarily mean more urine. The bladder collects and stores urine, but is not responsible for increased urine output induced by hypertension.
This answer choice contradicts our prediction. If we have more reabsorption, that’s going to mean decreased urine output. Think about it, there’s less water exiting the water via urine if it’s just reabsorbed. This answer choice actively contradicts what I went over in my breakthrough, and answer choice A was more neutral. I am going to stick with answer choice A as my superior answer choice because of this.
The collecting duct collects filtrate coming from nephrons, but filters water out to make urine more concentrated. If we increase permeability, we’re going to have increased reabsorption of water. Same problem as answer choice B.
I said during the breakdown: increased urine output induced by hypertension means increased glomerular filtration rate. That’s exactly what this answer choice is saying, and that’s exactly what we’re looking for in our question stem. This is a more direct answer choice that’s closer to that breakdown. We can eliminate answer choice A for not properly addressing the question being asked. We’re left with our correct answer, answer choice D: Increased glomerular filtration rate.
If restriction of blood flow to the kidneys (by placing clamps on the renal arteries) resulted in an immediate but small increase in blood pressure, followed by the gradual development of severe hypertension, which hypothesis would these results best support? This boils down to deciding if restriction of blood flow to kidneys and development of severe hypertension supports Hypothesis A or Hypothesis B.
We have clamps on the renal arteries, meaning we’re artificially preventing normal amounts of oxygen-rich blood from reaching the kidneys. Kidneys can no longer filter waste and remove excess fluid properly. That’s when the kidney’s blood pressure control mechanism comes in. The kidney will have to use hormones to regulate blood pressure and water balance. How does that happen? The renin-angiotensin-aldosterone-system: RAAS.
The kidneys secrete renin directly into circulation that results in the formation of angiotensin II. Angiotensin II causes blood vessels to constrict, resulting in increased blood pressure. Angiotensin II also stimulates the secretion of aldosterone from the adrenal cortex. Aldosterone causes tubules of the kidneys to increase reabsorption of sodium and water into the blood. That increases the volume of fluid in the body, and also increases blood pressure. That’s why we have the immediate increase in blood pressure, followed by gradual development of severe hypertension. That’s consistent with Hypothesis B, which says increased pressure should act to initiate an effective corrective reflex involving the kidneys. That’s what we’re seeing here, so I’m looking for an answer choice that talks about Hypothesis B.

This answer choice is technically true, but that resistance is so strong that normal amounts of blood are not reaching the kidneys, and we have decreased blood pressure. This isn’t a case where the vessel restricts partially, and systemic blood pressure increases, we’re actually dealing with a full clamp.

The reasoning for the answer choice is true. The clamps caused the kidneys to receive less blood. But how does that support hypothesis A? Hypothesis A focuses on vessel disease and vasoconstriction increasing blood pressure. Not the case when the clamps are minimizing blood coming in and reducing blood pressure in the kidneys.

This answer choice is consistent. We talked about RAAS and how the kidneys respond to the decreased blood pressure. Blood pressure slowly, but surely increases because of the effective corrective reflex of the kidneys. This is the best answer choice so far and we can eliminate answer choices A and B.
Volume of body fluids is going to increase as the kidney’s corrective reflex happens. Blood pressure and volume are actually increasing to compensate for the blockage. That means this answer choice contradicts what was said in the passage and what I said in my breakdown of the question. That means we can eliminate this answer choice and we are left with our correct answer, answer choice C: Hypothesis B, because the kidneys were responding to decreased glomerular blood pressure.

If blood pressure doubled and the resistance to blood flow increased by 50%, the amount of blood pumped by the heart would have. In other words, we’re doing a math problem where blood pressure doubles and resistance to blood flow increases 50%. Let’s solve for the amount of blood pumped. We can use the equation laid out when the author explains Hypothesis A.

We’re told this system illustrates the basic relationship between blood pressure (P), flow rate of blood from the heart (cardiac output or CO), and vascular resistance to the flow of blood (VR): P = CO × VR.
I’ll write everything out here, then show the solved problem down below. We’re going to double blood pressure, P. We’re going to increase resistance blood flow, VR, by a factor of 1.5, or 50%. We isolate cardiac output. We can divide both sides by 1.5 x VR. Now we have 2P / 1.5 VR = CO.
Simplifying the fraction on the left side gives us 1 1/3 P/VR = cardiac output. Meaning cardiac output increases by 1/3 in this situation.

This was a math problem where we did no rounding or estimating. There were no unit changes and we only dealt with ratios and fractions. We solved for the increase in cardiac output: we said cardiac output increases by 1/3. That means we can pick the answer choice that matches our calculation: answer choice A.

According to Hypothesis A, enhanced activity of which of the following basic muscle types would be most likely to cause hypertension? We’re going to pull up part of the passage here. The author mentions the cause of vasoconstriction in relation to muscles during the explanation of Hypothesis A:

We’re focused on something later in the paragraph. It says: an increase in systemic vascular resistance caused by factors such as vessel disease or enhanced muscle tone in the vessel walls (vasoconstriction) may be the major cause of systemic hypertension. We’re going to identify the type of muscle in vessel walls. That’s also known as vascular smooth muscle. Smooth muscle is involuntary muscle that’s found in the intestines, throat, uterus, and most importantly for our question: blood vessel walls.

Striated refers more to the structure of the muscle. For example, cardiac muscle is striated. But smooth muscle, which is our prediction, is non-striated. We can likely eliminate answer choice A as soon as we have a better option. This answer choice contradicts what I said in the breakdown and it contradicts our general knowledge.

This is the answer choice we’re looking for. Enhanced activity of the muscle in vessel walls can cause hypertension. That muscle is smooth muscle. That means we’ve found an answer that is superior to option A. Answer choice B is our best option at this point.

Cardiac muscle tissue is found only in the heart. That’s where cardiac contractions pump blood throughout the body and maintain blood pressure. That’s not the muscle we’re focused on here. We’re focused on smooth muscle-the involuntary muscle found in blood vessel walls.

Multinucleated muscle usually refers to skeletal muscle fibers which are under voluntary control. Smooth muscle cells will have a single, centrally located nucleus. That means we’re left with our correct answer, answer choice B: Smooth muscle.

What process would be most disrupted by an inflammation of the colon? In this question we’re dealing with a situation in which there are colon issues, so we’ll go over the functions of the colon and large intestine to see what process would be most affected. In the passage, he author mentions the large intestine, and specifically the colon, are affected by inflammatory bowel disease. What do we know about the large intestine? It’s a tube-like structure that absorbs water from the undigested food received from the small intestine. The colon specifically extracts water and mineral salts from undigested food and stores waste material. The colon also contains bacterial flora that produce vitamins.

This is a broad answer choice, but technically still true. The colon is near the end of the digestive system and we still have absorption of water and salts. We can hold on to answer choice A for now, we’ll see if we have any of our more specific functions in the other answer choices.
This is a better answer choice than option A, because it’s getting into a more specific function. The colon doesn’t absorb a lot of nutrients, but there is still some taking place. Digestion itself isn’t disrupted significantly like we saw in answer choice A, so we’ll keep answer choice B for now. We can eliminate answer choice A. It was too extreme.
This answer choice is similar to answer choice B, and it’s more specific than answer choice A. But what’s the big difference between answer choices B and C? Absorption of water is the primary function of the colon and large intestine in digestion. This is going to be a better answer than answer choice B. Answer choice C is superior.
This isn’t a main function of the colon. We already mentioned the main function as extracting water and mineral salts from undigested food. Secretion of digestive enzymes would be a better answer choice if we were dealing with other parts of the digestive tract. We can eliminate answer choice D. We’re left with our best answer, answer choice C, the primary function of the colon: Absorption of water.

Normally the immune system avoids attacking the tissues of its own body because. In other words, we’re going to explain why we don’t constantly have autoimmune responses. When antibodies are formed in a person’s own body against self-antigens, those are autoantibodies. Autoimmune diseases result from this immune response. The immune system mistakes some part of the body as a pathogen and attacks its cells. If that’s not ideal, then what do we want to happen instead? We want to suppress the activation of any T-cells or autoimmune responses. That allows the body’s immune system mechanism to still stay active, but there’s no issue with attacking the tissues of its own body.

This would be a great option, but is this actually what happens? It would be great if the immune system could recognize only foreign antigens, but instead that doesn’t happen. Instead the body has to suppress the activation of T-cells to prevent attacking of the body’s own cells.

This is factually incorrect. The reason autoimmune diseases are a thing is because the body makes self-antigens. The immune system recognizes these antigens, and in autoimmune diseases, we have an exaggerated response. We can eliminate answer choice B for not being factually correct.

This is another method which would be great in theory, but isn’t how the body works in practice. The body doesn’t change its antibodies to be specific to only foreign antigens. Again, if it did, we wouldn’t have to worry about autoimmune diseases. We can eliminate this answer choice also.

This answer choice is more reasonable, and its similar to our prediction. The body has to suppress cells that are specific to self-antigens. That prevents an immune response, and autoimmune diseases are avoided. We can keep this answer choice, and we can eliminate answer choice A. Answer choice A would be a great function if it were true. Instead we’re sticking with our correct answer, answer choice D: it suppresses cells specific to the body’s own antigens.
An ulcer that penetrated the wall of the intestine would allow the contents of the gastrointestinal tract to enter. In other words, We have an ulcer that penetrates the wall of the intestine. The contents of the GI tract can now enter _____ where the contents wouldn’t have entered before this ulcer.
This is almost like a standalone question that’s tangentially related to the passage. We will utilize our general knowledge of intestinal anatomy. Peritoneum is the membrane that lines the abdominal cavity and covers the GI organs. Penetration and perforation of the wall of the intestine allows for contents to enter the peritoneal cavity. That’s the space between the parietal and visceral peritoneum. Ideally, in a real-life setting there’s an inflammatory response, and the perforation is treated before the contents of the GI tract enter the peritoneal cavity. But when that’s not the case, we have to deal with issues caused by the contents entering the peritoneal cavity.
Perineum is a region in the pelvis between the thighs. It’s bound by the scrotum and anus in males, and the opening of the vagina and the anus in females. Just going by human anatomy and the location of the ulcer, this is going to be an incorrect answer choice. The most likely reason this answer choice was included is because it sounds like what we predicted the correct answer to be.
This answer choice matches what I mentioned in the breakdown of the question. I said we have penetration of the wall of the intestine. That means the contents can enter the space between the parietal and visceral peritoneum, which is the peritoneal cavity that lines the GI tract. Even if we didn’t know which answer choice was correct between options A and B, just going by the general location is enough to eliminate answer choice A.
Pleural cavity is the space between the visceral and parietal pleura of the lungs. For this question, we’re dealing with the GI tract, not the lungs. We definitely wouldn’t want the contents of the GI tract to hit the pleural cavity, but it’s also not what’s happening in the question. We can eliminate answer choice C as well.

The lumen of the intestine is part of the GI tract. That’s where we have numerous villi protruding into the lumen and absorbing the products of digestion. In other words, we want the contents to be in the lumen of the intestine, so we can eliminate answer choice D. We’re left with our correct answer, answer choice B: peritoneal cavity.

If the genetic and autoimmune theories of inflammatory bowel disease are true, then the gastrointestinal antigen being targeted by the immune system is probably on. What’s the initial takeaway here? The author prefaces the question by saying the genetic and autoimmune theories are true. Given that, where is the location of the GI antigen being targeted by the immune system? We can reference the passage once here to get a better sense of the answer we’re looking for.

This excerpt focuses on the genetic and autoimmune theories of inflammatory bowel disease. What does the passage say about antigens specifically? It says “An antigen in the body, perhaps in the digestive tract, is recognized as foreign by the immune system. This antigen may then stimulate the body’s defenses to produce an inflammatory response that continues without control.” Let’s give some background on antigens also. The cells that make up the body’s tissues and organs are covered with antigens. So, in general, we’re looking for an answer that talks about cell surface. These antigens can stimulate the body’s immune response. We’ll go through the four answer choices and utilize our general knowledge as necessary.

Think back to my breakdown of the question. I said antigens are going to be on the surface of cells, not on chromosomes. This answer is factually incorrect.

Same as answer choice A here. Antigens are going to be on the surface of cells and tissues, not part of DNA segments.

Similar to answer choices A and B here. We’re not expecting antigens to be part of nucleic acids, but rather on the surface of cells or tissues.

This answer is consistent with how antigens work. We said we’re looking for an answer that mentions antigens found on the surface of cells, tissues, and organs. Answer D is going to be our correct answer. The GI antigen being targeted by the immune system is probably on the surface of the proteins encoded by the genes for the disease. Maybe even more than we initially expected, this was a predominantly content-reliant question.

The fact that there appears to be a genetic component to inflammatory bowel disease, but that it does not show clear Mendelian inheritance ratios suggests any of the following, EXCEPT. As always, be careful about the wording here. The question is saying: the fact that there aren’t Mendelian inheritance ratios suggests any of the answer choices EXCEPT one answer. No need to go back to the passage for this question because the question stem already tells us everything we need to know from the passage. There appears to be a genetic component, but no clear Mendelian inheritance ratios
Mendelian refers to the inheritance of traits through genes and was originally proposed by Gregor Mendel. Mendel states that a cell holds a gene containing two alleles, with one allele inherited from each parent. This means that only two alleles, one dominant and one recessive, could exist for a given gene. The way the question is worded, we’re going to find an answer choice that’s actually consistent with mendelian inheritance.
This answer choice opposes the breakdown of the question. We’re looking for an answer choice that’s consistent with Mendelian Inheritance. We said when we’re talking about Mendelian inheritance we’re dealing with one dominant, and one recessive allele. Incomplete penetrance would mean people with the mutation for inflammatory bowel disease don’t actually have symptoms or features of inflammatory bowel diseases. The condition has incomplete penetrance.

This is similar to answer choice A. We don’t have clear Mendelian inheritance when we have limited expressivity. Limited expressivity means signs and symptoms of the disease can occur in individuals with the same diseases. So inflammatory bowel disease symptoms could be seen differently in different patients. That’s not indicative of Mendelian inheritance.

This is also not consistent with Mendelian inheritance. A polygenic disease means the disease is caused by the action of more than one gene, not consistent with only two alleles: one dominant and one recessive for a given gene. So far, no answer choices jump out.

This answer choice describes Mendelian inheritance. Mendel stated that a cell holds a gene containing two alleles, with one allele inherited from each parent. Two alleles, one dominant and one recessive, can exist for a given gene. That’s what this answer choice is saying. That means it matches what the test-maker was asking in the question stem. The genetic component does not show clear Mendelian inheritance ratios. That means we won’t be able to simply say the gene for the disease is recessive. That would imply clear Mendelian inheritance ratios. We can eliminate answer choices A-C, none of those demonstrate clear Mendelian inheritance ratios. We’re left with our best answer, answer choice D: the gene for the diseases is recessive.

The finches observed by Darwin on the Galapagos Islands are an example of adaptive radiation. In order to set up conditions that would produce adaptive radiation, it would be necessary to place members of: This standalone question is in a format that AAMC likes to use. We get some information that’s tangentially related to answering the question, but we can ultimately use our knowledge of the content to answer. For example, to answer this question we need to know about adaptive radiation. We’re given an example of adaptive radiation, but is it necessary to know details about the finches or Darwin to answer this question? Probably not. Just like a lot of passage-based questions rely almost exclusively on your general knowledge instead of information from the passage, this is just adding more information to the question that you may or may not end up using.
In adaptive radiation, organisms will diversify from an ancestral species into new forms based on their environment. Like the name suggests, these organisms will adapt based on need and fit to an environment, and enhance their reproductive potential. They will also adapt to their surroundings to better survive.
If an environment is constantly changing, there might not be enough time to observe these changes. Additionally, if the species is placed into one environment, we would not be able to see different organisms diverge and evolve independently. We’re looking for a better answer that takes this adaptation into consideration.
This answer choice is consistent with what we’re looking for in our breakdown. Think about the visual I added above. That ancestor finch diverges and the future organisms will adapt based on need. By placing these finches into different environments, the finches will adapt based on need and to better survive. This is a better answer than answer choice A in order to actually see adaptive radiation happen.

This answer choice starts with observing multiple species which is not the point of adaptive radiation. We’re looking at a single species and noticing how it may adapt as the environment changes. Answer choice B remains a superior option.

This is similar to answer choice C. We want to start with a single species and observe changes as it adapts to the environment. We don’t want to observe multiple species in the same environment. We can stick with answer choice B as our best option.

Hemophilia, a disease in which the time required for blood to clot is greatly prolonged, is determined by a sex-linked gene. Suppose a man with normal blood clotting marries a woman with normal blood clotting whose father was a hemophiliac. If this couple has three sons, what is the probability that hemophilia will be transmitted to all three of them?
First, let’s think about sex-linked genes. These are genes that are located on the sex chromosomes. Sex-linked genes generally refer to genes on the X chromosome in humans. Because males only have one X chromosome, they only inherit one allele of sex-linked genes, making them more susceptible to diseases caused by sex-linked genetic mutations. Males having only one X chromosome is going to be key to answering this question, as well as the mother being a carrier (we know the mother is a carrier because her father was a hemophiliac. The mother got the one X chromosome from her father).
We can draw out our Punnet square. We have a carrier mother (XXh) and an unaffected father (XY):X
Xh
X
XX
XXh
Y
XY
XhYNote what I highlighted here. That’s the only affected offspring in the square, but we have to be careful about what the question is asking. We want to know the probability hemophilia is transmitted to three sons (given the couple has three sons). We’re only worried about the males in the Punnet square. Of the two males, one is affected, so each son has a 50% chance of being affected. Three sons with a 50% chance means ½ x ½ x ½ total, or 1/8 probability that hemophilia is transmitted to the three sons of the couple.
Looking at our answer choices, answer choice A corresponds to our calculated value. Answer choice B would be correct if we were looking for the probability of any offspring being affected. Answer choice D is the probability each individual son is affected, not three sons. Answer choice C incorrectly adds together the 1/8 probability three times. We’re left with our correct answer, answer choice A.

 

The above graph represents an action potential recorded from the cell body of a neuron. What type of ion movement is causing the depolarization of the neuronal membrane at the time denoted by the arrow? This question relies 100% on knowing your content. Make sure you know what an action potential graph looks like, and what each of the parts represent. To make things easier for you, I’ve added the visual below. We can look at our answer choices and note which one corresponds to the depolarization of the neuronal membrane at the time denoted by the arrow in our question stem.

This answer choice matches what we’re seeing in the visual above. We have voltage-gated sodium channels open and sodium is moving into the neuron down its concentration gradient. This supports our breakdown, but we’ll keep comparing and see if we can find any superior answer choices.
This is as simple as looking closely at our visual. Sodium ions are not being moved out of the neuron via active transport, so we can eliminate this answer choice.
This is slightly premature. Soon the potassium ions will be moving, but that’s not the cause of the depolarization taking place. Answer choice A still remains superior.
This is similar to answer choices B and C. This just comes down to knowing and utilizing the action potential graph. Answer choice A is our best answer.

A sequence near the 3′ end of bacterial 16S ribosomal RNA (rRNA) base-pairs with a sequence called the Shine–Dalgarno sequence in the ribosome binding sites of prokaryotic mRNAs. Given that the sequence in the 16S rRNA is

3′ UCCUCCA 5′

what is the mRNA sequence of the Shine–Dalgarno sequence that most strongly binds the ribosome? One thing you always have to be aware of with the MCAT is the sequence of RNA or DNA presented to you. Sometimes it will be written 3’ to 5’ (like in this situation), while others you might get 5’ to 3’. Sometimes you’ll get a complementary sequence, sometimes you’ll get rRNA, and other times you’ll get DNA. Make sure you know exactly what’s being shown to you because that’s more than half the battle! In this case we have rRNA that’s written from 3’ to 5’ and we want to know the mRNA sequence that binds this sequence most strongly. That means a complement. Our answer is going to be:
5′ AGGAGGU 3′ ideally because that’s our complement. OR the test-maker might give us 3’ UGGAGGA 5’.

We have to be careful here because this is similar to our answer choice, but there is a T instead of a U, which is not what we want in this situation. Let’s keep looking for an answer that has uracil (because we’re looking for an mRNA sequence).

This answer choice matches our breakdown exactly. There’s not much subjectivity here, so good chance this is our correct answer.

This answer choice incorrectly attempts to bind C to U and A to G. That’s not the case for our complement. We’re still sticking with answer choice B.

This is the perfect example of what I warned you about. Make sure you know if a sequence is running from 5’ to 3’ or 3’ to 5’. In this case, the only difference between B and D is how the sequence is written. This is incorrectly written from 3’ to 5’, so answer choice B is our best and correct answer choice.

Which participant in the electron transport chain has the greatest attraction for electrons? To answer this question, we’ll consider the electron transport chain (ETC). We can pull up a visual to help us reason through our answers:

If this answer had instead said FADH2 this would be a more viable answer. We’re not dealing with the reduced form, however, so this is not a great option. Make sure to utilize the visual above for this question, but you should also commit it to memory for any future ETC questions.

This answer choice is similar to answer choice A. This is not a viable answer and we’d feel better if this answer choice instead said NADH. Neither answer choice A nor B are great options thus far.

This is something any professor or teacher that teaches the ETC likely says: oxygen is the final electron acceptor and has the greatest attraction for electrons. This is going to be our best answer choice so far, so we can confidently eliminate answer choices A and B.

Cytochrome C transfers electrons between complexes III and IV like we see in our visual, but like I mentioned, oxygen is going to be our best answer here. Oxygen is the final electron acceptor and has the greatest attraction for electrons, so we are going to pick answer choice C as our best answer.

In addition to the effects of estrogen deficiency, the most likely reason that more women than men suffer from osteoporosis is that women, compared to men, have.
We’re focusing on the reason more women than men suffer osteoporosis, but in addition to effects of estrogen deficiency. We’ll want to focus on levels of bone loss and formation in younger males and females-prior to seeing the effects of menopause and estrogen deficiency. The author mentions in the passage that postmenopausal women will have accelerated bone loss, and that can be due to estrogen deficiency. That’s the only mention the author makes of differences in bone mass between genders. However, we’re not focused on these older men and women because we want a reason more women will suffer from osteoporosis that doesn’t deal with estrogen deficiency.
From a younger age, males in general will have higher bone density and content, and also bone size. It’s true postmenopausal women will lose bone more quickly, but males also start with denser bones. That’s a big reason why, in general, women suffer more from osteoporosis than men. The combination of a lower bone density to start, and the eventual hormonal deficiencies.

This answer choice is consistent with our prediction. There aren’t any real anatomical or hormonal differences the author mentions, besides the accelerated bone loss in postmenopausal women. We want reasoning outside of estrogen deficiency, meaning we can focus on the differences in males and females at a young age. Like we said in our prediction, males will generally have higher bone size, and bone density. More dense bones have to support the generally higher body weight in males. The lower bone density in woman can lead to a higher rate of osteoporosis. Let’s keep this answer choice and keep comparing.

This answer choice is unreasonable. There aren’t actual anatomical differences like the number of vertebrae in males and females. We can eliminate this answer choice for being factually incorrect.

This is similar to answer choice B. There’s no mention in the passage about differences in mechanisms for calcium uptake. And in general, there aren’t big differences between males and females and mechanisms for calcium uptake. Bone tissue does contain a majority of the body’s calcium, but these mechanisms don’t differ, so we can eliminate answer choice C.

Similar to answer choices B and C. There aren’t such big differences in the underlying anatomy and functions of males and females. Calcium uptake and vitamin D production aren’t mentioned by the author. We don’t know of any differences between males and females in terms of vitamin D production either, so we can eliminate answer choice D. We’re left with our best answer, answer choice A: lower bone density.

Postmenopausal women receiving estrogen and progesterone therapy will most likely experience which of the following side effects? This answer is simply going to come from your general knowledge of estrogen and progesterone. We know postmenopausal women may have estrogen deficiency. That can be remedied through hormone replacement therapy. But excess estrogen can have adverse side effects if doses of progesterone aren’t also received. What would be the effects of this hormone therapy?
Progesterone and estrogen are two of the most important hormones in the female body. They work together to regulate the reproductive system, and female characteristics in general. Progesterone is often called the pregnancy hormone. It prepares the uterus for implantation, and if pregnancy doesn’t occur, progesterone levels decreases and that leads to menstruation. Both hormones also lead to the development of secondary sex characteristics, so supplementation would also influence that.
This answer choice contradicts my breakdown as estrogen and progesterone therapy would do the opposite of atrophy breast tissue. Both lead to the development of secondary sex characteristics. Breast development’s reliant on both estrogen and progesterone, in addition to other hormones, so we’re not liking this answer choice for now.

This is similar to answer choice A. Progesterone and estrogen will actually have the opposite effect. Both answer choices contradict our breakdown, so we won’t pick one answer choice over another just yet.

This answer choice is similar to what we’re looking for. We said that progesterone helps regulate menstruation and levels fluctuate depending on if there is implantation or not. Both progesterone and estrogen are important in maintaining the uterine cycle, so when women supplement with these hormones, the varying levels mean menstruation will resume. This is the only answer choice consistent with our general knowledge and question breakdown so we can eliminate answer choices A and B. Both have an opposite effect to what we’re expecting.
This answer choice is not entirely relevant or expected to happen. Lactation is controlled by different hormones. Two main hormones are involved: prolactin and oxytocin. High prolactin levels stimulate milk production, and oxytocin triggers the cells to squeeze the milk from the breast. This isn’t what we’re seeing in our question stem or when we have supplementation with estrogen and progesterone.
Answer choice D is not an obvious or direct contradiction like we saw in answer choices A and B, but lactation is mostly controlled by prolactin and oxytocin, not estrogen and progesterone. Ultimately, we pick answer choice C instead of answer choice D because answer choice C directly answered the specific question proposed in the question stem.
Production of which of the following hormones will be inhibited by the administration of dietary calcium to prevent osteoporosis? For this question we can focus on the effects of calcium on bone formation and resorption. Administration of dietary calcium means elevated calcium levels in the blood. What does that mean? There’s no need for bone resorption because calcium levels will already be increased.
Osteoporosis decreases a significant portion of a person’s bone mass-in the passage we’re told roughly a third to half of bone mass. Osteoporosis is usually the result of deficiencies in diet like a lack of calcium, vitamin D, or vitamin C. Let’s go over some of the key players. Vitamin D functions to stimulate calcium absorption and to enhance the effect of parathyroid hormone. Other agents like parathyroid hormone and calcitonin also play a role. Parathyroid hormone promotes bone resorption through osteoclast activity. Calcitonin does the opposite by decreasing bone resorption. Usually older people are affected, and more women than men are affected. That’s because of anatomical and hormonal differences. Ultimately, there’s more bone resorption than bone formation, and that becomes problematic because of the decreased bone mass.
Growth hormone wasn’t mentioned explicitly in the passage, and we didn’t cover it in the breakdown. It’s not involved directly in regulation of blood calcium levels or affected significantly by dietary calcium. Logically, growth hormone would promote additional calcium to be used for bone formation, so we wouldn’t see its action inhibited.
Calcitonin decreases bone resorption and increases bone reformation. How does it do that? By working to decrease calcium levels in the blood, and using or storing calcium in bone. This answer choice is a direct contradiction, so we can eliminate answer choice B for being worse than answer choice A. Answer choice A is still superior, just because there’s no direct correlation or mention in the passage about growth hormone and calcium levels.
Thyroid hormones are two hormones released by the thyroid gland and regulate metabolism. The test-maker might be trying to confuse us here, because thyroid hormone sounds similar to parathyroid hormone. This is going to be similar to answer choice A though. It’s not a directly correct or incorrect answer choice. The author doesn’t mention thyroid hormone, and there’s no close tie between calcium levels and thyroid hormone.
This answer choice is something the author actually mentions in the passage. Parathyroid hormone encourages the activity of osteoclasts. What do osteoclasts do? Break down bone cells and release more calcium into the blood. If we administer dietary calcium, we have increased calcium levels in the blood already. What does that mean? There’s no need for parathyroid hormone to be working. Increased calcium levels will inhibit production of parathyroid hormone.

A man is treated with low doses of an estrogen analogue to destroy an estrogen-responsive adrenal tumor. Compared to an age-matched control (no estrogen treatment), this patient’s chances of developing osteoporosis will most likely be. We do want to recall a key point from the passage: accelerated postmenopausal bone loss in women appears linked to estrogen deficiency. Treating a male with an estrogen analogue likely won’t any effect. Why is that? Because males don’t suffer the same sharp postmenopausal decrease in estrogen levels. Males don’t have high estrogen levels to begin with, so there’s minimal risk of developing estrogen deficiency.

This answer choice contradicts our prediction. Treating a male with estrogen should have a minimal effect on the risk of developing osteoporosis. That being said, estrogen supplementation decreases the chances of women developing osteoporosis. It’s unlikely that risk would increase in males.
This would likely be the answer choice if we were focused on women instead of men. But like we said, men don’t have elevated estrogen levels to begin with, so there’s not the same estrogen deficiency in older men as there is in postmenopausal women. This is still a better option than answer choice A which would be untrue for women also.
This sounds like our breakdown and it’s consistent with our thinking. Low doses of an estrogen analogue are not going to affect the man’s risk of developing osteoporosis. We want to keep this answer choice, and we can eliminate answer choice B. That answer choice was applicable to women, not men.
First part of this answer choice sounds right, but the author also provides reasoning. Is there any reason to believe the disease will appear at an earlier age? The passage doesn’t mention this and there’s no mention of estrogen affecting osteoporosis risk in males in general. That means we’re left with answer choice C: approximately the same.
Osteoblasts, which form bone, and osteoclasts, which resorb it, work together to cause continuous bone remodeling. In a person suffering from osteoporosis, which of the following combinations of changes in the activity of these cell types will most likely occur? This can be boiled down as simply asking: What happens to osteoclast activity and osteoblast activity in osteoporosis? That means we’ll go through the mechanism of osteoporosis, and how osteoclasts and osteoblasts work. Osteoporosis is a decrease in a person’s bone mass. Osteoblasts create and mineralize bone; osteoclasts reabsorb bone tissue. The test-maker actually tells us this in the question stem also. Logically, when we have a decrease in bone mass, we have more resorption than formation. Said differently, more osteoclast activity and less osteoblast activity.
Increased osteoblast activity would mean more bone formation. Decreases osteoclast activity means less bone resorption. Both directly contradict our breakdown of the question, and the question stem itself.
We’re not seeing an increase in activity of both cell types. That would theoretically mean no net change in bone mass. In reality, osteoporosis is a decrease in bone mass. This is slightly better than answer choice A because it mentions increased osteoclast activity, so we can eliminate answer choice A because it was a direct contradiction.

This is similar to answer choice B. A decrease in activity in both cell types could theoretically be a net zero change. If there was a larger decrease in osteoclast activity, we might actually see an increase in bone mass. We’re going to hold on to both answer choices B and C for now.

This answer choice is consistent with our breakdown. Decreased osteoblast activity means less bone formation. Increased osteoclast activity means more bone resorption. Are these two always true? Is there a permanent decrease in osteoblast activity and permanent increase in osteoclast activity? Not necessarily. That’s why the author mentions, this will most likely occur. We know we have a net decrease in bone mass, and this is the only answer choice consistent with that decrease.

A male taking excess testosterone may become infertile because of reduced spermatogenesis. According to Figure 2, this could result directly from. We’re focused on two different things: First, how might increased testosterone reduce spermatogenesis? And how might this cause infertility? We can reference Figure 2 from the passage. We may need to use our general knowledge to explain why there’s reduced spermatogenesis or infertility.

We have Figure 2 here for our reference. Question stem says there’s excess testosterone, and reduced spermatogenesis. First, what happens when we have excess testosterone? We have negative feedback, meaning less GNRF release from the hypothalamus. Less GNRF means less stimulation of the pituitary to synthesize and release FSH and LH. What does that ultimately mean? FSH acts directly on the Sertoli cells to promote and maintain spermatogenesis. Remember, Sertoli cells protect and provide nutrients for developing sperm. We have excess testosterone, so the LH side of our figure should be fine. There’s no need for Leydig cells to stimulate the production of more testosterone. But what we do have to worry about, is the reduction in FSH synthesis and release.

An increase in inhibin concentration would inhibit FSH release. FSH acts on Sertoli cells to promote and maintain spermatogenesis. But would the increased testosterone actually cause an increase in inhibin concentration? Increased testosterone would actually inhibit GNRF release, meaning there would be less inhibin produced downhill of this process.
Based on what I just said, this would be the result of increased testosterone, but a reduction in inhibin concentration would actually mean less negative feedback and an increase in FSH activity. FSH acts directly on the Sertoli cells to promote and maintain spermatogenesis. Spermatogenesis wouldn’t be reduced. That being said, we can still eliminate answer choice A because increased testosterone wouldn’t correlate to increased inhibin concentration.

We already discussed that FSH concentration would be reduced. So, we know this is a potentially correct answer choice. Would a reduction in FSH concentration lead to reduced spermatogenesis? We said FSH acts directly on the Sertoli cells to promote and maintain spermatogenesis. Increased testosterone would mean a reduction in FSH concentration, and ultimately reduced spermatogenesis. We’re liking this answer choice. That means we can eliminate answer choice B because that wouldn’t correlate to reduced spermatogenesis.

This answer choice is a viable option also. Increased testosterone would actually inhibit GNRF release. GNRF stimulates the pituitary to synthesize and release LH. That means we would have a reduction in LH concentration. The only issue here is LH acts on Leydig cells, not Sertoli cells. We wouldn’t see reduced spermatogenesis so we can eliminate answer choice D as well. We’re left with our correct answer, answer choice C: a reduction in FSH concentration.

The cell type in the male reproductive system that is most analogous to the female ovum is the. This is a standalone, content-based question that’s disguised as a passage-based question. We can do an overview of the female ovum, then match with a cell type in the male reproductive system. The ovum is the female gamete. Gametes are haploid cells used in sexual reproduction; they’re formed during meiosis, which consists of one round of chromosome replication and two rounds of nuclear division. In females that’s the haploid, mature ovum. Fusion of two gametes can form a zygote.
In males we saw the equivalent in Figure 1.We go from spermatogonium to primary spermatocytes, haploid secondary spermatocytes, spermatids, and finally spermatozoa. Our answer is going to be spermatozoa. I’ve pulled up Figure 1 above, just because it can be helpful to look at Figure 1 while we go through our answers, just for reference.

Spermatogonium are undifferentiated, diploid cells that will go through mitosis and eventually get to spermatozoa. For now, this doesn’t match what we said in our breakdown. We want a superior answer choice.

Just like spermatogonia, the primary spermatocyte is diploid. We have first meiotic division that gets to a haploid cell called the secondary spermatocyte, and eventually additional division to get to spermatozoa. This answer choice is better than answer choice A because it’s closer to our predicted answer of spermatozoa. We can eliminate answer choice A and keep comparing.

This is once again closer to what we’re looking for in our correct answer. Like we just said, we have haploid secondary spermatocytes formed following the first meiotic division of primary spermatocytes. We have secondary spermatocytes undergo a second meiotic division and form spermatids. Spermatids ultimately differentiate into spermatozoa which is our prediction. That means this answer choice is going to be closer to our prediction than answer choices A and B.

This answer choice is consistent with our predicted answer. Spermatozoon is the answer we’re ultimately looking for and the male reproductive counterpart of the female ovum. This matches our breakdown and Figure 1 from the passage. We’re left with our correct answer, answer choice D: Spermatozoon.

Which of the following hormones is(are) directly required for spermatogenesis?

Luteinizing hormone (LH)
Follicle-stimulating hormone (FSH)
Inhibin
Testosterone
In other words, which hormones listed are directly required for spermatogenesis. Emphasis here on the directly. All of these hormones work together and may tangentially affect spermatogenesis, but some will affect spermatogenesis directly. We’re going to look at Figure 2 from our passage with our 4 Roman numeral options and see if/how each hormone is related, or required for spermatogenesis.

We’re going to focus on Sertoli cells here. Sertoli cells protect and provide nutrients for developing sperm, and promote spermatogenesis. The author mentions two key points in the passage about Sertoli cells.
First point: Testosterone acts on the Sertoli cells to promote maturation of sperm.
Second point: FSH acts directly on the Sertoli cells to promote and maintain spermatogenesis.
Those are two of our four choices. Luteinizing hormone acts on Leydig cells, not Sertoli cells. Inhibin is produced by Sertoli cells, but inhibits FSH release. We’re going to stick to the two options the author explicitly mentions: Testosterone and FSH.
We’re going to find an answer choice that includes testosterone and FSH. That’s only answer choice C. As always, with these Roman numeral questions if you’re sure one option is correct or incorrect, use that to your advantage. For example, just by eliminating option I, we can eliminate two of the four answer choices. Take any opportunity you can to get the numbers in your favor.

Which of the following statements correctly describes the distinction between the exocrine and endocrine portions of the testis?
This is something that you may have considered during your readthrough of the passage, but you can rely on some key points from your general knowledge. Exocrine glands secrete substances into a duct, while endocrine glands will secrete products directly into the blood. So main difference is ducts. For our passage specifically: exocrine portion consisted of seminiferous tubules and endocrine portion consisted of Leydig cells.
This answer choice isn’t factually correct. Exocrine portion isn’t limited to secreting only peptides and the endocrine portion isn’t limited to secreting steroids. Instead, we want an answer dealing with ducts versus no ducts, and it should be factually correct.

This answer choice matches our breakdown, and it isn’t specific to this passage only. Rather I explained the difference in definitions between exocrine and endocrine. It’s the difference between releasing products into ducts versus releasing products into the blood. We can eliminate answer choice A. It’s too limited and extreme of a definition, and it’s factually incorrect.

This is also similar to answer choice A. It’s too extreme in limiting the exocrine and endocrine portions. We’re not limiting the substances that are secreted from each gland, rather where the secretions will go: either directly into the bloodstream or into a ductal system. We can eliminate answer choice C.

Product of the endocrine portion is the product of Leydig cells. Leydig cells produce testosterone which targets male sexual organs, secondary male sex characteristics, different aspects of testicular activity through negative feedback, and Sertoli cells. Testosterone’s target is not limited to seminiferous tubules. We can eliminate answer choice D. We’re left with our correct answer, answer choice B: The exocrine portion releases its products into ducts; the endocrine portion releases its products into the blood.
Uric acid enters the urine both through filtration and secretion in the kidney. The process of filtration of uric acid in the kidney takes place in the. This is essentially a standalone question that’s related to the topic in the passage. We’re focusing on the excretory system and specifically filtration in the kidneys. Where does filtration take place in the kidneys?
We’re going to talk this out and focus on filtration. Let’s go over the glomerulus and Bowman’s capsule. The glomerulus is the site in the nephron where fluid and solutes are filtered out of the blood to form a glomerular filtrate. Bowman’s capsule is a sack at the beginning of the tubular component of a nephron. It performs the first step in the filtration of blood to form urine.
That’s consistent with our breakdown. I said the glomerulus is the site in the nephron where fluid and solutes are filtered out of the blood. That’s exactly what we’re looking for here.
Loop of Henle is a structure in a kidney’s nephron that connects the proximal convoluted tubule to the distal convoluted tubule. The loop of Henle, the proximal and distal tubules, and the collecting ducts are sites for the reabsorption of water and ions. Not a big player in filtration like the glomerulus-we can kill answer choice B.

We touched on this a second ago. Distal convoluted tubule has an important role in the absorption of many ions and in water reabsorption, but not so much in filtration. That means we can eliminate choice C because we have a superior option.

This answer choice is involved in the resorption of sugar, sodium and chloride ions, and water from the glomerular filtrate, but not filtration itself like answer choice A. That means we can eliminate answer choice D and we’re left with our correct answer, answer choice A: glomerulus.

Colchicine most likely relieves gout symptoms through what mechanism? The author briefly talks about colchicine, so we can flip back to the passage. And remember, I bring these parts of the passage back up for the sake of demonstration. I want you to be able to follow along with my thought-process. Adding an excerpt or a figure helps me communicate with you more effectively. What I’m not trying to advocate for here is re-reading the entire passage or even flipping back to the passage more often than is absolutely necessary.

The passage says Another drug used to treat gout is colchicine, an inhibitor of microtubule reorganization. We can contrast this with allopurinol which deals more directly with uric acid levels, by inhibiting xanthine oxidase. Colchicine is instead an inhibitor of microtubule reorganization. Microtubules are hollow tubes that are composed of tubulin proteins. They help the cell transport materials within itself, and resist shape changes. Colchicine is most likely focusing on the inflammatory response in joints, not on the concentration of circulating uric acid directly.

This answer choice is focusing on uric acid diffusion through membranes. Microtubules focus more on cell shape and transport within the cell, not diffusion through membranes. Another thing we want to note, we said we’re focusing more on the inflammatory response, not on the circulating uric acid.
This answer choice is focused on the inflammatory response in joints. Phagocytosis is what leads to hyperactivation of inflammatory cells. Inhibiting phagocytosis would also happen by resisting shape change, which is a function of microtubules. By inhibiting phagocytosis, we don’t see the negative effects of the inflammatory response. We like answer choice B which means we can eliminate answer choice A because it contradicted our breakdown.

This answer choice again addresses the uric acid component of gout, but not the inflammatory response. Microtubules deal with cell transport and cell shape, not uric acid crystal formation. Answer choice C, just like answer choice A, contradicts what was said in the breakdown.

Another answer choice that doesn’t match our definition of microtubules or address the function of microtubules. Colchicine doesn’t affect pH, or the activity of PRPP synthetase. This answer choice doesn’t address anything the author brings up in the passage. It also isn’t consistent with our prediction. We can also eliminate answer choice D for contradicting the breakdown of the question. We’re left with our correct answer, answer choice B: Inhibition of leukocyte phagocytosis of uric acid crystals.

What nitrogenous base would promote the formation of uric acid crystals in gout?
The author mentioned in the passage, “Uric acid is formed by the breakdown of purines to xanthine.” Ultimately, this answer is going to come from our general knowledge, and we’re going to identify the nitrogenous bases that are purines.
Thymine, uracil, and cytosine are classified as pyrimidines. Adenine and guanine are purines.If you ever forget which nucleobases are classified as which, I’ve always used a few short phrases.
Pure As Gold (purines = adenine, guanine) – 2 rings
CUT The Pie (pyrimidine = cytosine, thymine, uracil) – 1 ringed
Cytosine is a pyrimidine.
Uracil is also a pyrimidine.
This answer choice matches our prediction. Guanine is a purine, so we can eliminate our other two answer choices so far because those were both pyrimidines.
Thymine is also a pyrimidine. We’re left with our correct answer, and the only listed purine: guanine.

In the patient described in the passage, the likely genetic basis of the increased levels of uric acid is a mutation. We’ll revisit the third paragraph in the passage, because that will give us a clue about which mutation is causing the increased uric acid.

We have the third paragraph of our passage here where we go over the patient with recurrent gout. Key points here:
Excreted uric acid levels were 3x normal. That means we have elevated uric acid in the body.
Increased levels of 5-PRPP (which along with glutamine, synthesizes uric acid)
Normal levels PRPP synthetase, but 3x normal enzyme activity (in the body)
Outside the body, enzyme works normally and at normal levels. So, we just have an issue with enhanced activity in the body itself.
Biggest takeaway is we’re seeing enhanced enzyme activity, and not more enzyme itself. This enhancement is happening in the body, and not when the enzyme is studied in vitro.

Feedback regulation of an enzyme happens when a product of the reaction binds to an allosteric site on the enzyme, and affects its catalytic activity. Catalytic activity is enhanced in the body by affecting the allosteric site. The fact that the enzyme works normally outside of the body is consistent with this answer. The enzyme itself isn’t affected, and works normally if the allosteric site is unaffected.
This answer choice directly contradicts what we just mentioned in our breakdown of answer choice A. An enzyme’s active site binds to the substrate. So, if we affect the active site of the enzyme itself, that would also alter enzyme activity in vitro: likely in a negative, or competitive way. Typically, when we have an increase in catalytic activity, that’s the result of binding to the allosteric site. So, we suspect an in vivo molecule affects the allosteric site. Answer choice B contradicts our breakdown.
If this were the case, we’d have increased levels of PRPP synthetase, which isn’t the case. We have normal enzyme levels, just increased enzyme activity in the patient. This answer choice contradicts what we learned in the passage.
This answer is similar to answer choice C. If this were the mutation, we’d have more PRPP synthetase. The author mentions there aren’t higher levels of the enzyme, just higher levels of enzyme activity. We can eliminate answer choice D as well. All 4 answer choices addressed the specific question being asked, but only one answer matches our breakdown and is consistent with what the author presented in the passage. That means we’re left with our correct answer, answer choice A: affecting an allosteric site of PRPP synthetase.
A drug that binds to tubulin molecules of plant cells and prevents the cells from assembling spindle microtubules would most likely cause the resulting plants or plant cells to have: We can break this question down a little bit at a time. We have a drug that binds to tubulin molecules of plant cells. We know from our content that tubulin proteins compose microtubules. The author explicitly tells us this drug prevents the cells from assembling spindle microtubules and wants us to know the effect. Microtubules help the cell resist compression, provide a track along which vesicles move through the cell, and pull replicated chromosomes to opposite ends of a dividing cell. Presumably one of these functions would be most affected and will be our best answer.
Preventing cells from assembling spindle microtubules is not going to change the actual genes in plant offspring. Let’s see if we can find a better answer choice.
As I mentioned, microtubules function to pull replicated chromosomes to opposite ends of a dividing cell. If we don’t have the normal segregation of chromosomes into daughter cells, we have unequal chromosomes in the daughter cells. If we mess with the normal separation during anaphase, we expect more than the normal two sets of chromosomes. This is a better answer than answer choice A.

The test-maker is trying to get you to pick this answer because it talks about cell walls and we’re dealing with plants and plant cells. However, there is no tubulin in the cell wall, so this answer choice is factually incorrect. Answer choice B remains the best option.

Microtubules help the cell transport materials within itself and resist shape changes. They also make up elements of flagella and cilia, but the effect on movement is not as significant as the effect on pulling replicated chromosomes to opposite ends of dividing cells.

During the repolarization phase of an action potential in a neuron, which of the following is generally true of the voltage-gated channels that cause that action potential? This is exclusively a content question that’s testing how well you know the action potential graph. Let’s add that in here and go through our answer choices. We’re concerned with repolarization.

Looking at our graph, we know that voltage-gated K+ channels are open and Na+ channels are closed. This answer choice contradicts what we’re seeing in the graph. Let’s look for a better option.

This answer choice is only half correct. We expect voltage-gated K+ channels to be open, but Na+ channels are closed. This is still better than answer choice A which was fully incorrect. Answer choice B is now our best option.

This is similar to answer choice B because it is half correct. We know that voltage-gated K+ channels are open and Na+ channels are closed during repolarization.
This matches the action potential graph exactly. During repolarization, voltage-gated K+ channels are open and Na+ channels are closed. We can stick with our best option: Answer choice D.
An organism that causes a human disease is isolated and studied. Researchers conclude that the organism is a bacterium rather than a virus because the organism: To answer this question we’ll have to distinguish between bacteria and viruses. We want to note a characteristic or property of bacteria, but not viruses. This question is fairly open-ended, but there are certain key points AAMC tends to focus on more than others. Bacteria are living organisms that consist of a single cell that can make its own food, move, and reproduce. Viruses grow and reproduce inside host cells they infect. Bacteria are much larger than viruses, but their infections are typically localized. Viruses can infect a host and multiply quickly. Let’s look through our answer choices and see which options are consistent with these differences, or we may even consider some other differences.

It is possible the researchers were studying a bacterium, but there’s no way they would know it’s not a virus. Viruses can mutate over time, so this is not something unique to bacteria.

This is similar to answer choice A. Bacteria will not have a nuclear membrane, but this is not something unique to bacteria. Viruses will often pass the nuclear envelope barrier during infection of a host, but a lack of a nuclear membrane is not going to distinguish between a bacterium and a virus themselves.

Viruses contain a protein coat, or capsid. Seeing protein in its outermost covering would not be a sign of a bacterium over a virus. Let’s keep looking for a better answer.

This is only possible when we’re dealing with bacteria and not viruses. We said viruses grow and reproduce inside host cells they infect, but bacteria are living organisms that can live in many places because they can make their own food, move, and reproduce. Answer choice D is going to be our best answer choice.
Which of the following correctly pairs a cellular process with the location in which that process occurs in a prokaryotic cell? This is just a content question where we’re pairing location with a cellular process. The thing we want to be cognizant of is the fact that we’re looking at prokaryotic cells specifically. Prokaryotes are single-celled organisms that have no membrane-bound organelles.

This answer choice is consistent with what we know about bacteria. Normally transcription takes place in nuclei, but a lack of nuclei in prokaryotes means we have transcription in the cytoplasm instead. This is a good answer choice to start with, let’s keep comparing.

This answer choice lists a membrane-bound organelle which is not something that’s present in prokaryotic cells.

This answer choice also lists a membrane-bound organelle which is not something that’s present in prokaryotic cells. Answer choice A remains the best option.

This answer also choice lists a membrane-bound organelle which is not something that’s present in prokaryotic cells. We can eliminate answer choices B-D. We’re left with our correct answer, answer choice A.

Glucose is labeled with 14C and followed as it is broken down to produce CO2, H2O, and ATP in a mammalian liver cell. In theory, during this process the label will be detectable: To answer this question, we want to track the labeled glucose as it’s broken down. Quick glance at our answer choices also shows that both location and order are important here. We’ll have to consider the breakdown of glucose which is as simple as glycolysis in the cytoplasm followed by the Krebs cycle which takes place in the mitochondria. While I do recommend you know the details about these pathways, we have to know what this question is asking. All we’re asked is where we’d be able to detect 14C, but not any additional, specific details. The only answer choice in this situation that’s factually correct is answer choice D: first in the cytoplasm, then in the mitochondria.
 Why is the Ames test for mutagens used to test for carcinogens? To answer this question, we’ll explain how mutations are related to cancer. The author mentions the Ames test is used in the initial screening of carcinogenic compounds. Why is that? Because it provides a good indication of the mutagenic characteristics of many chemicals. Let’s go through some background and tie these terms together.
Mutations contribute to genetic variation within species. They can be inherited and have a positive effect, or they can disrupt regular gene activity and cause diseases like cancer. Cancer’s caused by mutations occurring in several growth-controlling genes. Substances that cause mutations are mutagens. Mutagens are agents that cause mutations in the DNA of cells; and carcinogens, specifically, are agents that lead to cancer. The author implies in the passage that mutations are often also carcinogens, and that’s why the Ames test is so effective.
We just mentioned in our breakdown that mutagens are often carcinogenic already. Salmonella doesn’t transform the mutagens. Rather the mutagens will cause the bacteria to back-mutate.

This answer choice is similar to our breakdown and what the author mentions in the passage. Mutagens are often carcinogens. The Ames test provides a good indication of the mutagenic characteristics of chemicals which ends up also being a good initial screening of carcinogenic compounds. We can keep this answer over answer choice A. Like we just mentioned, the statement in answer choice A is backwards and contradicts what we read in the passage.

This answer choice is out of scope. Salmonella could contain oncogenes, but does that explain the relationship between mutagens and carcinogens? This answer choice doesn’t explain why we use the Ames test to test for carcinogens, so we can eliminate answer choice C.

There’s nothing in the passage that suggests this statement is true. We’re not told salmonella’s RNA is able to distinguish between carcinogens and mutagens. In general, that’s not something we expect to be true. We can also eliminate answer choice D. We’re left with our correct answer, answer choice B. The Ames test for mutagens is used to test for carcinogens because most mutagens are also carcinogens.

The passage indicates that when Salmonella have back-mutated, they. This answer is going to come from the passage. We can go back and see what the author mentions about the bacteria that back mutate.

The passage says During the Ames test, the suspected carcinogen is added to a histidine-deficient growth medium. If the chemical is a mutagen, some of the bacteria will back-mutate, and a visible colony will form.
We add the suspected carcinogen to a histidine-deficient growth medium. Normally, the special strain of bacteria we use during this test can’t grow without histidine. The mutagen causes bacteria to back-mutate. That strain normally can’t survive on a medium without histidine, but now we’re seeing visible colonies form. These bacteria that had back-mutated have regained the ability to produce their own histidine.

This answer choice isn’t something that’s mentioned in the passage. When the salmonella back-mutates, it regains the ability to produce histidine and survive. There’s no mention of pigment-the colony being visible is just reference to bacterial colonies forming in the growth medium, not pigment.
This answer choice is consistent with our prediction and what the author mentions in the passage. Previously this mutated salmonella couldn’t survive without histidine, but now these bacteria that back-mutated. They regained the ability to produce their own histidine.

Another answer choice that’s not mentioned in the passage. The author doesn’t talk about salmonella metabolizing the carcinogen in the presence of light. Only thing we know is when the bacteria mutate, colonies will form. Meaning the bacteria can now survive despite the lack of histidine in the medium; the bacteria can produce their own histidine. We can eliminate answer choice C and answer choice A. Answer choice B was explicitly mentioned in the passage, and is the superior answer.

Back-mutated Salmonella actually regain the ability to make histidine. Salmonella that haven’t back-mutated can’t survive without histidine. This answer choice also contradicts our prediction and our passage. We can eliminate answer choice D as well. We’re left with our correct answer, answer choice B: the strains are capable of synthesizing histidine

Which of the following best explains why bacterial colonies formed on Plate IV in Figure 1? Before doing anything, we’ll look at Figure 1, and specifically Plate IV (Plate 4).

We have plates 1-4 here and we can see colonies formed on all 4 plates. We’re focused on Plate 4, which was testing purified air. In theory, this pure air shouldn’t have had any bacterial colonies form.
The best guess we can make is there are mutations that occurred that caused bacterial colonies to form on Plate 4. Why might I make that guess? Because the author says the bacteria lack a mechanism for DNA repair. That means chances of mutation occurring are higher. That would be one of the only explanations here. There shouldn’t be any mutagens in purified air, and there shouldn’t be any mutagens in the plates themselves.

This isn’t likely considering the author tells us plate 4 contained purified air. We don’t expect any mutagens in the air, and we don’t expect any of the bacteria to back-mutate. This answer choice contradicts our breakdown.

This is similar to answer choice A. Plate 4 is our standard. We shouldn’t have any mutagens in the purified air, or in the agar. These mutagens come from chemicals like the air in cities A-C. The visible colony in plate 4 more likely happened because of mutations and a lack of DNA repair.

This answer choice is consistent with our breakdown. The author says the bacteria lack a mechanism for DNA repair. That means chances of mutation occurring are higher, and there aren’t going to be corrections of any mutations. That gives us an explanation for why bacterial colonies formed on plate 4, even though we didn’t expect that to happen. We can kill answer choices A and B for contradicting our prediction and the experimental setup the author mentioned in the passage.

This answer choice is a direct contradiction of the passage. The author mentions the special Salmonella strains lack a mechanism for DNA repair. We can kill this answer choice also. We’re going to stick with our correct answer, the one that matches our prediction. Answer choice C: spontaneous mutations occurred.

Cancer cells most likely have an abnormality in their. To answer this question, we’re going to relate mutations and cancer, and where abnormalities come in to cause cancer. Mutations are changes to the base sequence of DNA which are not intended. These mutations can lead to genetic variation. They can be inherited and have a positive effect or they can disrupt regular gene activity and cause diseases, like cancer. We touched on this earlier in our workshop. Cancer’s caused by mutations occurring in several growth-controlling genes. That means we’re looking for an answer choice that mentions abnormalities in DNA, AKA a mutation.

This answer choice matches our initial breakdown of the question. I said mutations are abnormalities in DNA, and that is what’s happening in cancer cells. We have mutations occurring in growth-controlling genes, and that’s why we have uncontrolled growth of abnormal cells.
rRNA is known as Ribosomal RNA. It’s an important component of ribosomes, and used to build ribosome structures to read mRNA, and create proteins from amino acids. We might have elevated rRNA expression in cases of cancer. But we want a cause, which we said comes from abnormalities in DNA, not rRNA. We’re still going to stick with our superior answer choice, answer choice A.

Mitochondria are the powerhouse of the cell and produce ATP. Mitochondria are necessary for cancer cells to thrive. But they’re not the cause of cancer. Instead, mitochondrial abnormalities correlate to the inability to utilize oxygen and a lack of energy. This is inconsistent with our breakdown and the question stem. Answer choice A is still the superior answer choice.

Lysosomes are membrane-bound organelles that contain digestive enzymes for breaking down part of the cell, and material that’s taken into the cell by phagocytosis. Cancer cells are not going to have issues in lysosomes, but rather we have abnormality in DNA. If anything, research has actually been done to use lysosomes to kill cancer cells. But that’s outside the scope of the exam. We can also eliminate answer choice D-it’s inferior to our best option, answer choice A: cancer cells most likely have an abnormality in their DNA.

For Treatment 2 to be effective, the antibody must. This is a question that relates directly to the passage. The author walks us through Treatment 2 and the role of the antibody. Let’s revisit Treatment 2 and Figure 1 at the same time.

Treatment 2 says Administration of an antibody that blocks the attachment site for the endotoxin-binding protein complex on macrophages. I actually added a red X through this point in our cascade. What exactly is happening here? Treatment 2 functions to prevent macrophage activation and the subsequent, exaggerated inflammatory response. We can see that in our figure. We won’t have the release of cytokines right below the macrophage in our figure. We’re not continuing down our flowchart which eventually would have led to hypotension and shock. We’re looking for an answer that circles around the idea that the antibody needs to prevent macrophage activation, which should prevent the subsequent, problematic inflammatory response.

This is the opposite of our prediction and what the author mentions in the passage. Treatment 2 functions to prevent macrophage activation and the subsequent inflammatory response. Not to stimulate macrophage activation.
This also contradicts our prediction. By preventing macrophage activation, we don’t have the subsequent release of proinflammatory messenger peptides. Meaning no cytokine release. That would mean the opposite of stimulating T-cell production. The antibody doesn’t directly affect T-cell production. And if the antibody works properly, we expect T-cell production to decrease. Still, this doesn’t directly contradict our breakdown like answer choice A.

This answer choice sounds better than answer choice B, just because T-cell production may decrease if we don’t have macrophage activation and subsequent cytokine release. Is the antibody actually inhibiting T-cell production directly? Not necessarily, but this is still our best answer choice so far. That’s inconsistent with what we expect to happen.

This answer choice matches our breakdown. The author explicitly mentions the antibody will block the attachment site for the endotoxin-binding protein complex on macrophages. That ultimately prevents macrophage activation-and means no cytokine release or exaggerated inflammatory response. Answer choice D is the best of the four answer choices.
What is a danger of using the drug in Treatment 1? To answer this question, we’re going to focus on the mechanism of Treatment 1, and how general anti-inflammatory drugs can be dangerous.
Very quick recap of Treatment 1: To prevent the exaggerated inflammatory cell response from endotoxin infection, anti-inflammatory drugs are used. Why are we using these drugs? To counter the inflammatory cell response. In the passage, we went through the cascade of events that cause septic shock (including cytokines and leukocyte proliferation).
This drug blocks the inflammatory cell response, which sounds good in theory, but there may be some downside. The immune system is vital to fighting pathogens. What this general anti-inflammatory drug does is also block some other necessary inflammatory cell responses. Using a general anti-inflammatory would suppress vital inflammatory cells, and inhibit the fighting of other infections.

This answer choice sounds similar to our prediction. What does endogenous mean? Internal, or within. So, this general anti-inflammatory drug would prevent antibacterial activities that are normally crucial to fighting pathogens and staying healthy. That’s why the drug in Treatment 1 is potentially dangerous.

Would platelet count be affected by anti-inflammatory drugs? We’d see a bigger influence on white blood cells than we would on red blood cells. Platelet count would not fluctuate to dangerous levels like this answer choice is insinuating. Answer choice A is still the superior answer, and more directly answers the question.

This answer choice is similar to answer choice B, so our reasoning here is going to be nearly identical. We’d expect to see a bigger detriment on our white blood cell activity and count than we would on red blood cells. A decrease in red blood cell count would be dangerous if that were the case. We can eliminate answer choice C.

Given the circumstances and what the drug is trying to prevent, answer choice D is actually listing something favorable. The drug is meant to combat septic shock syndrome, which is the result of hypotension. An increase in blood pressure would combat this hypotension and actually be a positive result. We can kill answer choice D. It directly contradicts what the question stem is looking for, and instead offers us a positive aspect of the drug. We’re left with our correct answer, answer choice A: It may decrease endogenous antibacterial activities.

Fever in septic shock leads to which of the following compensation mechanisms? In other words, we’re going to focus on the body’s compensatory mechanisms whenever we have a fever. Fevers are usually caused by an infection or illness. Temperatures in the body can raise to over 100 degrees Fahrenheit, or over 38 degrees Celsius. There are other symptoms that accompany a fever, but the inflammatory response that we deem the fever itself is elevated temperature. The elevated temperature can actually be helpful to enhance immune defenses, but the question is asking how the body can compensate for this elevated temperature. The hypothalamus will stimulate vasodilation, which allows more blood flow to the skin and release of heat from the body. Additionally, there’s also sweating, which cools the skin as the sweat evaporates. Ultimately, the body’s going to be using a combination of these two to compensate for the increased temperature.
This answer choice is consistent with our breakdown of a fever. This explains one of the main methods by which humans can compensate for elevated body temperature. Capillaries dilate, which allows for heat to dissipate through the skin. Let’s hold on to this answer choice.

Increased skeletal muscle activity isn’t expected, and it’s not a way the body traditionally cools itself down. In fact, increased skeletal muscle activity would actually generate more heat. Think of your muscles moving and increased muscle activity. That makes you warmer and it doesn’t cool you down. We can eliminate answer choice B because it contradicts our breakdown.

If we slow down respiration rate, we’re actually not allowing for heat to escape the body. Think of a dog panting when it’s warm. That helps release heat and bring body temperature down, and that’s the ultimate goal here. We can eliminate answer choice C because it has the opposite effect of what we’re looking for.
This answer choice is similar to answer choice C. I actually discussed sweating and relying on evaporation as a cooling method. We expect to increase fluid loss to compensate for the fever. That’s also a big reason why you’re always told to hydrate properly whenever you’re sick-because we want to account for the fluid loss. We can eliminate answer choice D because it also contradicts our breakdown. We’re left with our correct answer, our best answer, answer choice A: Dilation of capillary beds in the skin.
If the anti-inflammatory drug in Treatment 1 interfered with DNA replication, in which phase of the cell cycle would cells tend to be arrested? The drug in Treatment 1 interferes with DNA replication, and the cell cycle is arrested in a specific phase. We want to identify the phase of the cell cycle that corresponds to DNA replication. To answer, we’ll break down the phases of the cell cycle, and note any phases that deal with DNA replication.
The cell cycle has two major phases: interphase (G0, G1, S, G2) and the mitotic phase (M). Ultimately, the cycle produces two new daughter cells.
During interphase, the cell grows, and DNA replicates. In the mitotic phase, the replicated DNA and cytoplasmic contents separate, and the cell divides. We’re dealing with interference of DNA replication, so we’re focusing on interphase.
The G0 phase is a resting phase. The cell is either waiting for a signal to trigger the onset of G1, or cells that don’t divide might be in G0 permanently.
G1 phase is the growing phase. The cell grows and produces organelles. It has the option to divide or not divide.
Next is the S phase: the synthesis phase. This is where DNA replication occurs. That’s exactly what we’re looking for in our question. We have centrioles replicate and organize cell division.
G2 phase is when the cell replenishes its energy stores and synthesizes proteins necessary for chromosome manipulation. The cell is preparing for division. We have double the DNA and another increase in size.
After interphase is when the cell enters the mitotic phase, but that’s not going to be relevant to DNA replication.
I said the G0 phase is the resting phase. Ultimately, we want an answer choice that says S phase, or synthesis phase. That’s when DNA replication happens.
G1 phase is the growing phase. We have cell growth and production of organelles, but not yet to DNA replication.
This is the synthesis phase and the answer we were looking for. The S phase is where DNA replication occurs. If an anti-inflammatory drug interferes with DNA replication, the cell cycle would be arrested in the S phase. DNA replication can’t happen properly, so the cell is stuck in this phase. We can eliminate answer choices A and B. We went through those phases in the breakdown, and neither one corresponds to DNA replication like the S phase.

G2 phase is more preparation for division. We already had DNA replication in the S phase. We won’t even properly make it to the G2 phase because we expect the cell cycle would be arrested in the S phase, so we can eliminate this answer choice. The anti-inflammatory drug in Treatment 1 interferes with DNA replication. The cell cycle would be arrested in answer choice C: S phase.

Pulmonary arterial blood differs from the aortic blood because it has. We want to distinguish between pulmonary arterial blood and aortic blood, and the author mentions a pressure difference between the two in the passage. The author mentions mean pulmonary arterial pressure is much lower than mean aortic pressure. What’s the difference between the two? Blood returns to the lungs through the pulmonary arteries. They’re carrying deoxygenated blood from the right ventricle to the lungs. Once blood gets to the lungs, it becomes oxygenated, and disposes of carbon dioxide.
Now on the other hand, we’re also asked about aortic blood. That deals more with systemic circulation. Oxygen-rich blood from the lungs arrives via pulmonary veins. The blood passes to the left ventricle where it is pumped out through the aorta, the major artery of the body, taking oxygenated blood to the organs and muscles of the body.
What are the big differences between the two? Pulmonary arterial blood pressure is lower than aortic pressure. Pulmonary arteries carry deoxygenated blood to the lungs. That means we have a low concentration of oxygen, and a higher concentration of carbon dioxide. Once blood gets to the lungs, it’s oxygenated, and disposes of carbon dioxide. Aortic blood is oxygenated, and has lower levels of carbon dioxide. Mean aortic pressure is also higher than mean pulmonary arterial pressure. Take a look at the image showing blood circulation below, and use it to visualize as you’re answering any related questions.
We addressed oxygen and carbon dioxide levels during our breakdown, but what about pH? That’s going to be affected by the carbon dioxide. At higher concentrations, carbon dioxide reduces pH. The carbon dioxide reacts with water and forms carbonic acid. That carbonic acid dissociates to release proton (H+) and bicarbonate ion. More protons means more acidic, or a decrease in pH. Looking at answer choice A again, all 3 parts of this answer contradict our breakdown. I said we expect less oxygen and more carbon dioxide in the pulmonary arterial blood. Additional carbon dioxide meant more acidic, or lower pH.
This answer choice is slightly better because we have more CO2. But we said that would be lower pH. Our ideal answer was also less oxygen, not more oxygen. We can still eliminate answer choice A. All 3 parts contradicted our breakdown. Answer choice B is less incorrect.
This answer choice is consistent with my breakdown. Pulmonary arterial blood is lower in oxygen. Once blood gets to the lungs, it becomes oxygenated, and disposes of carbon dioxide. We said pulmonary arterial blood has higher levels of carbon dioxide, and that causes a lower pH. We can now eliminate answer choice B because contradicted our breakdown.

First part of the answer starts out correct, but when we get to carbon dioxide and pH, those contradict what was said in our breakdown. We expect pulmonary arterial blood is higher in carbon dioxide, and a lower pH. We can also eliminate answer choice D. We’re left with our correct answer, answer choice C: less O2, more CO2, and lower pH.

Contraction of the diaphragm results in a. To answer this question, we’ll use basic anatomy to decide what happens when the diaphragm contracts. We’ll tie that into breathing and the lungs. We can use a quick diagram to take us through the process.

One thing we want to note, on both sides we’re going to have negative pressure. The thoracic cavity always has a slight, negative pressure. That helps keep the airways of the lungs open. Let’s look at the picture on the left. That’s where we’re going to focus-contraction of the diaphragm. During the process of inhalation, the lung volume expands as a result of the contraction of the diaphragm and intercostal muscles. We have expansion of the thoracic cavity, and a decrease in pressure relative to the environment.
Upon exhalation, the lungs recoil to force the air out of the lungs. The intercostal muscles relax, returning the chest wall to its original position. The diaphragm also relaxes, moving higher into the thoracic cavity. We see the upward arrow in the figure on the right. We have an increase in pressure within the thoracic cavity relative to the environment. Air rushes out of the lungs due to the pressure gradient between the thoracic cavity and the atmosphere.

Second part of this answer choice matches our breakdown, but we didn’t specifically discuss intrapleural pressure. Intrapleural pressure (IPP) is the pressure within the pleural cavity. The author mentions in the passage, IPP is always less than atmospheric pressure. When the thoracic cavity increases in volume during inspiration, that intrapleural pressure lowers even further, meaning it becomes more negative. Why does the pressure decrease? Because volume increased. The two are inversely proportional in a closed system. Even when exhaling, that pressure still has to be slightly negative relative to atmospheric pressure to ensure the lungs don’t collapse. We’re liking this answer choice for now.

First part of our answer choice matches our prediction. But expiration would result in a less negative IPP, not a more negative IPP. Also, we have the diaphragm contracting, we’re expecting inhalation, not expiration. Also, remember to be careful about these pressures and noting which one is more negative relative to the other. Answer choice A is still superior.

This answer choice is similar to answer choice B. We can’t have inspiration correspond to more positive IPP. The two are going to be opposites. Inspiration corresponds to more negative IPP like answer choice A. Expiration corresponds to more positive IPP. We can eliminate answer choice C.

This answer choice is better than answer choices B and C. We matched expiration with a more positive IPP. We already broke down the correlation between volume in the thoracic cavity and IPP. But the issue with this answer choice, is we’re dealing with contraction of the diaphragm. We established that corresponds to inspiration. Once we eliminate answer choice D, we’re left with our correct answer, answer choice A: more negative IPP and inspiration.

In comparison with the wall of the right ventricle of the heart, the left ventricular wall is. To answer this question, we can go through the functions of the ventricles, and what we know about the differences between the two. We’ll do a quick overview of pulmonary and systemic circulation. We’ll refer to a figure of the heart.

Start with pulmonary. Right atrium receives deoxygenated blood from the superior vena cava. This deoxygenated blood passes to the right ventricle. The right ventricle pumps the blood through the pulmonary arteries to the lungs for re-oxygenation.
Systemic circulation starts in the left atrium. Oxygen-rich blood from the lungs arrives via the pulmonary veins. The blood passes to the left ventricle where it’s pumped out through the aorta. This oxygenated blood is taken to the organs and muscles of the body.
Let’s see what differences we can spot. Where is blood being pumped out of each ventricle? Right ventricle it’s just being pumped to the lungs. Left ventricle it’s being pumped to all parts of the body. The wall of the left ventricle has to be much thicker than the wall of the right ventricle.

This answer choice is interesting. We said the left ventricular wall is thicker, which contradicts this answer choice. But look at the reasoning here is “generates a higher pressure when it contracts.” That has to be true for the left ventricle because that blood has to go throughout the body, not just get pumped to the lungs. The first part of the answer itself contradicts our breakdown, but the reasoning here is consistent with the left ventricle.

This answer choice completely contradicts our breakdown of the question. Left ventricular wall is thicker and it generates a higher pressure when it contracts. We can eliminate answer choice B. Answer choice A is still superior because it correctly explains the left ventricular wall generates a higher pressure when it contracts.

This answer choice starts out great-we said left ventricular wall is thicker. But the reasoning here matches the part of answer choice A that we liked. It generates a higher pressure when it contracts. Why might that be? Because that blood has to make it to the rest of the body, not just the lungs like the blood pumped out of the left ventricle. We can hold on to answer choice C, let’s eliminate answer choice A now because it incorrectly said the left ventricular wall is thinner.

First part of this answer choice is consistent with our breakdown, but second part here is a contradiction. The wall is thicker, and generates a higher pressure when it contracts. That higher pressure is out of necessity. We can eliminate answer choice D also and stick with our correct answer, answer choice C: Thicker and generates a higher pressure when it contracts.
If an artery that supplies blood to a lung lobe was blocked but ventilation to the lobe was unaffected, how would alveolar gas partial pressures change? In this case, there is no blood flow to a lung lobe, but ventilation works fine. What happens to alveolar gas partial pressures? We’ll focus on the consequences of there being no blood flow into the lung lobe, but still having normal ventilation. No blood being supplied to the lung lobe means there’s no elimination of carbon dioxide from the bloodstream to the lungs. So relative to what is normal, we would have decreased partial pressure of carbon dioxide in the alveoli. Normally, carbon dioxide passes from the blood into the alveoli, and then it can be exhaled. We don’t have blood getting to the alveoli in this situation. There’re also no red blood cells being oxygenated. Normally, blood travels away from the lungs through the pulmonary veins and into the rest of the body. That means in this case, we’d expect decrease carbon dioxide, and increased oxygen.

This answer choice contradicts what I said in the breakdown. We do expect partial pressure of oxygen to increase, but there’s no carbon dioxide exchange because of the blockage in the artery. This answer is only half true, so let’s see if we can find a better one.

This answer choice is similar to answer choice A. We would have a decrease in partial pressure of carbon dioxide, but oxygen levels wouldn’t also increase. This answer is just the opposite of answer choice A. Unfortunately, both answers are half right so it’s tough to eliminate one over the other just yet.

This answer choice matches our initial breakdown of the question. Normally, carbon dioxide passes from the blood into the alveoli, and then it can be exhaled. We have blockage of an artery that supplies blood to a lung lobe, so we don’t have the usual blood getting to the alveoli in this situation. No carbon dioxide exchange, means we have decreased levels.
No red blood cells being oxygenated either, but ventilation is still working properly. That means we have increased partial pressure of oxygen. We can eliminate answer choices A and B. We already mentioned both answers were half wrong, and both contradicted our prediction.
This answer choice is the opposite of our prediction. Carbon dioxide levels in the arterial blood might be higher, but alveolar partial pressure of carbon dioxide are going to be much lower. We can use the same reason we used to eliminate answer choices A and B to also eliminate answer choice D. We’re left with our correct answer, answer choice C: PO2 would increase and PCO2 would decrease.

In humans, cholesterol is a precursor to: We’re going to need to give some background on cholesterol and use that to get a sense of the answer we’re looking for here. Most lipid hormones are derived from cholesterol. The primary class of lipid hormones in humans is the steroid hormones. The big examples of steroid hormones include estradiol and testosterone. Other steroid hormones include aldosterone and cortisol, so we’re looking for something along these lines as our best answer.

While we are looking for a hormone of some sort, this might not be our best answer. There are three major types of hormones: lipid hormones, amino acid-derived hormones, and peptide hormones. Insulin is a peptide hormone, but we’re looking for a lipid hormone. Let’s see if we can find a better answer.
Glycogen is a form of energy storage, but not a lipid hormone, or even a hormone for that matter. Answer choice A is still our best answer because it listed a hormone (albeit peptide hormone).
This answer choice matches our breakdown and we can see how structurally similar testosterone is to cholesterol in the image I have included. Lipid hormones are generally structurally similar to cholesterol, which is exactly the case here.
DNA is also not a hormone, so we can eliminate this answer choice without much consideration. Cholesterol is not a precursor to nucleic acids. We can stick with our best answer choice here, answer choice C: testosterone.
One characteristic common to arteries, veins, and capillaries is the:
Endothelial cells are specialized cells that allow the permeability of selective materials through the walls of the blood vessels. They also allow movement of the white blood cells from the blood vessels to the site where immune defense is required to fight against foreign particles. Additionally, they help release certain molecules from the blood at the site of the injury to assist in the formation of clots. This is a viable answer that’s found in the walls of all three types of blood vessels.
This is a good answer if we’re dealing with veins which are larger in diameter. These valves would not be able to function correctly in arteries and capillaries. Answer choice A is the best option.

This ties a bit into what we mentioned in answer choice B. Not all arteries can dilate or constrict to regulate blood flow. Alternatively, we have endothelial cells in all three types of blood vessels, so answer choice A remains superior.

This describes something that is true of capillaries, but we need something that’s true of arteries and veins as well. We can eliminate answer choice D. We’re left with our correct answer, answer choice A.

The pancreas produces which of the following substances for the digestive system? This question boils down to knowing your content, and specifically knowing details about the pancreas. The pancreas contains enzymes such as amylase, lipase, ribonucleases and various types of proteins that help to digest carbohydrates, lipids, RNA, and proteins respectively. For the sake of the MCAT, we want to remember that most of the digestive enzymes are produced in the pancreas.

The bile duct releases pancreatic juice and bile into the small intestine, but we’re focused on bile salts. We want to know if bile salts are produced by the pancreas. Bile salts are actually produced by the liver and are stored in the gallbladder, which means we will likely eliminate this answer choice.

This ties into answer choice A. Bile salts will act as an emulsifier based on their makeup. However, we said bile salts are produced by the liver, so we can eliminate this answer choice.

Like the name suggests, gastric juices are secreted by the stomach glands and are not produced by the pancreas. Another answer choice that doesn’t match our breakdown and what we’re looking for.

Here we go, this is the answer we’ve been looking for! Digestion is a topic where there are a lot of moving parts (literally!), and many of them are related. Think back to our breakdown, we said most of the digestive enzymes are produced in the pancreas and that’s exactly what this answer choice entails. That means we can confidently eliminate answer choices A-C. We’re left with our correct answer, answer choice C.
Embryonic mouse cells divide every 10 hours at 37oC. How many cells would be produced from an egg after three days? This is a simple math problem where we have to be aware of our units. Let’s break down what this means. If we start with a single cell, after 10 hours we would have two cells (the initial cell divided after 10 hours). After another 10 hours (20 hours total from when we started initially), those two cells will divide and we will now have 4 cells. This happens for 3 days. How many hours are in 3 days? 72 hours (24 hours/day x 3 days).
We can break down the number of cells after every 10-hour interval:[]We’re looking for an answer choice that is roughly 128 cells (3 days is 72 hours, not only 70, but it’s a good estimate). Looking at our answer choices, each range of values is spread far apart. The only one that’s close to our estimation is answer choice B. 128 eggs is between 50 and 500 cells.
Which of the following describes a primary function of the myelin sheath? Let’s go through some background information here before we jump into our answer choices. Most mammalian nerves are surrounded by a whitish, fatty layer called the myelin sheath which we’re focused on for this question. We want to know its function. The myelin sheath insulates nerve fibers to prevent signal loss or crossing of signals. Additionally, myelin increases the speed of conduction in the axon. The myelin sheath has two primary functions: protection (physically and to prevent signal loss) and speed.
We went through the main functions of the myelin sheath and they did not involve providing nutrients to motor neurons. Let’s keep looking for an answer that’s consistent with our breakdown.
This discharge is regulated by a voltage-dependent calcium channel, not directly by the myelin sheath. Similar to answer choice A, this does not match the functions we came up with in our breakdown of the myelin sheath.
This is similar to answer choices A and B. We already broke down the main functions of the myelin sheath and it was not to guide dendrite growth and branching. Proper dendrite growth and branching is necessary, but it’s not the primary function of the myelin sheath.
At certain intervals along the axon, there are small breaks in the myelin sheath with exposed areas of axon membrane called nodes of Ranvier. You can see that in the visual I provided above. The myelin sheath allows action potentials to travel more quickly by jumping from node to node. This answer choice matches our breakdown and the function we mentioned for the myelin sheath. We can eliminate answer choices A-C and stick with our best answer: answer choice D.

Leave a Reply

Your email address will not be published. Required fields are marked *